Dorman: Emergency Medicine EOR Practice Exam (Rosh Review)

Pataasin ang iyong marka sa homework at exams ngayon gamit ang Quizwiz!

Which of the following structures is most likely to be injured in a knee (tibiofemoral) dislocation? A. Biceps femoris tendon B. Femoral vein C. Popliteal artery D. Tibial nerve

Correct Answer: (C) Popliteal artery Explanation: Tibiofemoral dislocations are serious injuries with a high rate of complications. They are usually due to high-energy trauma, such as a fall from a height or motor vehicle trauma, but can occur in obese patients who fall from a standing height. Approximately 50% of knee dislocations will reduce spontaneously prior to presentation to the emergency department, so it is important to obtain a history of the mechanism of injury and the position of the leg during and immediately after the injury. Dislocations are most commonly anterior or posterior but can also occur in medial, lateral, or rotatory directions. Physical examination findings include a joint effusion due to hemarthrosis, ecchymoses, recurvatum (hyperextension) of more than 30°, and laxity to testing of at least three out of the four major ligaments (anterior cruciate, posterior cruciate, medial collateral, and lateral collateral). X-rays are indicated to determine the direction of the dislocation. They may be normal if it has been reduced or may show subtle asymmetry of the joint spaces. Immediate closed reduction is indicated for dislocations that have not reduced spontaneously. Damage to the popliteal artery is the most common injury associated with knee dislocations and occurs in 40% of patients. The popliteal artery is anchored distally by the soleus muscle and tethered across the popliteal space, so it is vulnerable to injury. The presence or absence of pulses is not reliable for evaluating injury to the popliteal artery, so the ankle-brachial index should be measured in all patients with knee dislocations. If it is below 0.9, further studies, such as an arterial duplex ultrasound or a computed tomographic angiogram, should be performed. If it is 0.9 or higher, serial neurovascular examinations are indicated. The knee should be splinted in 20° to 30° of flexion to relieve tension on the nerves and blood vessels. Magnetic resonance imaging is indicated to evaluate the soft tissues once a vascular injury has been ruled out. Surgery to reconstruct the torn ligaments is usually delayed until swelling is resolved, but emergency surgery may be necessary for arterial repair. Complications of knee dislocations are frequent and include arthrofibrosis, peroneal nerve injury, compartment syndrome, instability, and deep venous thrombosis. The biceps femoris tendon (A) forms the lateral border of the popliteal fossa and is flexible and mobile, so it is less likely to be injured. The femoral vein (B) is located proximal to the knee and is unlikely to be injured in a knee dislocation. The tibial nerve (D) and the common peroneal nerve are branches of the sciatic nerve. While peroneal nerve injuries, usually neuropraxic injuries, are common, the tibial nerve has a straighter course in the posterior leg and is much less likely to be injured.

What is the difference between a Bakri intrauterine balloon catheter and an ebb Complete Tamponade System?

The ebb Complete Tamponade System has two balloons, one for intrauterine compression, and one for vaginal compression, while the Bakri intrauterine balloon catheter provides intrauterine compression only.

What is the dose of tamsulosin used to facilitate spontaneous stone passage?

0.4 mg once daily for up to four weeks

What is the bursal fluid white blood cell count that distinguishes inflammation from infection?

2,000 white blood cells per mm3

An 82-year-old man presents to the emergency department accompanied by his daughter with a complaint of altered mental status developing over the past four days. He is currently on aspirin therapy. An aide at his nursing home reported the patient had a mild fall when getting out of bed several days prior. He has complained of a headache, is disoriented on examination and demonstrates left-sided weakness. A noncontrast CT scan of the head is obtained. Based on the above image, what is the likely diagnosis? A. Epidural hematoma B. Left cortical infarction C. Subarachnoid hemorrhage D. Subdural hematoma

Correct Answer: (D) Subdural hematoma Explanation: A subdural hematoma is intracranial bleeding characterized by the development of a hematoma between the dura and the arachnoid membranes secondary to tearing of the bridging veins. It can be classified as acute, subacute, or chronic depending on the timing of the presentation. Subdural hematoma is demonstrated on head CT as a crescent-shaped collection of blood across the hemispheric convexity that deforms the surface of the brain. It most commonly develops from trauma to the head, such as a fall, motor vehicle collision, or assault. Elderly patients and those with a history of alcohol abuse are at increased risk due to the underlying cerebral atrophy. An acute subdural hematoma presents within 48 hours after onset, a subacute subdural presents within 3-14 days after onset, and a chronic subdural presents 15 or more days after onset. Presenting symptoms in a patient with an acute subdural hematoma include progressive neurological decline, symptoms suggestive of increased intracranial pressure (headache, vomiting, anisocoria, cranial nerve palsy), and coma. Patients with a more chronic subdural hematoma develop gradual cognitive impairment, headaches, and possible seizure. A CT scan of the head is the initial recommended imaging study in a patient with a suspected intracranial hematoma. Hematoma collection in the acute setting will appear hyperdense, while in the subacute and chronic setting it will appear hypodense. An acute subdural hematoma is a neurological emergency that may require surgical intervention to decompress the brain and evacuate the hematoma. Depending on the size and severity of symptoms at presentation, patients with smaller and chronic hematomas can be observed rather than undergo surgery. Anticoagulation should be reversed in the appropriate patient settings. Patients who develop elevated intracranial pressure who cannot undergo surgery may need other interventions including hyperventilation, hypertonic saline, or intravenous mannitol. Epidural hematoma (A) is incorrect. Epidural hematoma appears lenticular on imaging. Left cortical infarction (B) is incorrect. This would present as an area of ischemia and not collection of blood products. Subarachnoid hemorrhage (C) is incorrect. This would present with blood products within the subarachnoid and intraventricular spaces.

What is the Patient Health Questionnaire-9?

A self-evaluation tool that can be used to determine a baseline level of dysfunction in those with major depressive disorder and subsequently used to determine effectiveness of treatment.

What does the swinging flashlight test for?

Afferent pupillary defect: the affected pupil will not constrict to the same degree as the unaffected pupil when light is directly shone upon it, although consensual constriction will be within normal limits.

A 30-year-old man presents to the emergency department with a fever and cough for two days. Rales are heard over the right lower lobe. Which of the following exam findings supports the most likely diagnosis? A. Expiratory wheezing B. JVD C. Pretibial edema D. Tactile fremitus

Correct Answer: (D) Tactile fremitus Explanation: Pneumonia is an infection of the pulmonary parenchyma. The clinical presentation of pneumonia varies. In mild presentations, patients present with fever, cough, and shortness of breath. In severe cases, patients may present with sepsis and respiratory distress. Cough (with or without sputum production), dyspnea, and pleuritic chest pain are among the most common symptoms. Other symptoms may include chills, fatigue, malaise, and decreased appetite. Vital sign changes associated with pneumonia include fever, tachycardia, tachypnea, and hypoxemia. The physical examination findings associated with community-acquired pneumonia include increased work of breathing and adventitious lung sounds, including crackles and rhonchi. In addition, tactile fremitus, egophony, and dullness to percussion suggest pneumonia. Patients with severe disruption in alveoli gas exchange may present with hypoxemia. Pneumonia can be more subtle in elderly patients or immunocompromised patients as these patients are less likely to be febrile or have leukocytosis. In septic patients with pneumonia, hypotension, altered mental status, and organ dysfunction may be present. The diagnosis of pneumonia is confirmed with chest imaging. Chest X-ray is the most common study used. The initial step in the management of community-acquired pneumonia is determining the appropriate treatment setting. The most commonly used severity scores to help determine treatment setting are the pneumonia severity index and the CURB-65 (confusion, uremia, respiratory rate, BP, age ≥ 65 years) score. Patients with community-acquired pneumonia are initially treated empirically. However, most hospitalized patients undergo microbiologic testing to identify the causative pathogen, which helps to reduce broad-spectrum antibiotic use. Microbiologic testing may include blood cultures, sputum Gram stain and cultures, Streptococcus pneumoniae urinary antigen testing, Legionella polymerase chain reaction testing, and respiratory virus panels. The outpatient regimen for pneumonia varies according to the local resistance rate to Streptococcus pneumoniae, which is above 25% in the United States. The recommended outpatient regimen in the United States is either doxycycline monotherapy, respiratory fluoroquinolone monotherapy (levofloxacin or moxifloxacin), or a combination of a beta-lactam antibiotic (amoxicillin, amoxicillin-clavulanate, cefuroxime) and a macrolide (azithromycin). Hospitalized patients with community-acquired pneumonia are typically treated with either a respiratory fluoroquinolone or a combination of a beta-lactam and macrolide. In patients who have risk factors for methicillin-resistant Staphylococcus aureus, then vancomycin or linezolid is used. In patients with risk factors for Pseudomonas aeruginosa, piperacillin-tazobactam, cefepime, ceftazidime, imipenem, or meropenem should be used. Expiratory wheezing (A) is typically found with obstructive lung diseases, such as asthma or chronic obstructive pulmonary disease. Jugular venous distention (B) and pretibial edema (C) are each associated with volume overload states, such as congestive heart failure.

What medications can cause idiopathic thrombocytopenic purpura (ITP)?

Gold, quinidine, ranitidine, furosemide, and nonsteroidal anti-inflammatory drugs.

What is a Lhermitte sign?

An electric shock sensation that runs down the back and limbs with flexion of the neck, which is commonly seen in multiple sclerosis.

What are some common mechanisms of injury in knee dislocations?

Anterior dislocations may be caused by hyperextension, and posterior dislocations may be caused by a blow to the proximal tibia, such as would be seen in a dashboard injury in a car.

What is the MOA of atropine?

Atropine increases heart rate and improves atrioventricular conduction by blocking the parasympathetic influences on the heart.

What are the classic physical exam findings that may be seen in basilar skull fracture?

Battle sign, raccoon eyes, hemotympanum, and CSF otorrhea and rhinorrhea.

Postpartum Hemorrhage

Blood loss of ≥ 1,000 mL or bleeding associated with signs and symptoms of hypovolemia within 24 hours of birth regardless of route of delivery Most commonly caused by uterine atony PE will show an enlarged boggy uterus Management: Empty bladder Bimanual exam and uterine massage Oxytocin and additional uterotonics (e.g., prostaglandins) Tamponade (balloon or surgery)

Brown Recluse Spider

Brown violin shape on cephalothorax Southern midwestern United States Cytotoxin → local tissue destruction Antivenin (antivenom) unavailable Rx: supportive care

What is the most common cause of pneumonia in patients who are between the ages of 3 weeks to 3 months?

Chlamydia trachomatis

What condition is a porcelain gallbladder indicative of?

Chronic cholecystitis

A 42-year-old woman is being evaluated for acute cholecystitis. Which of the following diagnostic studies is most accurate for detecting cholelithiasis? A. Abdominal ultrasound B. Abdominal x-ray C. Computed tomography abdomen D. Magnetic resonance imaging abdomen

Correct Answer: (A) Abdominal ultrasound Explanation: Abdominal ultrasound is the best test to determine the presence of cholelithiasis. The majority of patients with cholecystitis have gallstones at the time of diagnosis. While acalculous cholecystitis can occur, it is not as common as gallstone-related cholecystitis. Risk factors for developing gallstones, and therefore cholecystitis, include being female, over the age of 40, overweight, and of child-bearing age due to progesterone causing biliary stasis. In addition, severe caloric restriction or weight loss surgery can precipitate gallstone creation. Patients will present with acute onset of epigastric or right upper quadrant pain with or without nausea, vomiting, or fever. While pain is classically described as colicky, most patients report constant pain. One distinguishing factor between gallstone disease, meaning symptomatic gallstones, and cholecystitis is the duration of abdominal pain. Gallstone disease presents with epigastric or right upper quadrant pain that resolves in four to six hours. In addition, vital signs, physical exam, labs, and imaging are expected to be normal with gallstone disease in contrast to cholecystitis. Acute cholecystitis is expected to have epigastric or right upper quadrant pain lasting longer than four to six hours and a positive Murphy sign on exam with or without tachycardia. Patients should not have jaundice or scleral icterus. Labs typically reveal leukocytosis with normal liver function tests. In addition to visualization of a gallstone, location of a gallstone in the cystic duct supports the diagnosis of cholecystitis. Gallbladder wall thickening, gallbladder distention, and pericholecystic fluid can also be visualized. Treatment of cholecystitis includes broad-spectrum antibiotics, intravenous fluid replacement, and cholecystectomy. Timing of the cholecystectomy will depend on the duration of symptoms. Ideally, laparoscopic cholecystectomy is performed within 48-72 hours of presentation. Advantages of performing a laparoscopic cholecystectomy early include shorter hospital stay and decreased nosocomial infections, however, it has a higher rate of needing to convert to an open cholecystectomy procedure due to inflammation and inability to completely visualize structures due to organ edema. Delayed laparoscopic cholecystectomy, performed several months after the acute cholecystitis has resolved, is associated with greater success of the laparoscopic approach, but patients are more likely to have complications of acute cholecystitis in the acute phase, such as gallbladder perforation. Those undergoing delayed cholecystectomy are managed with pain control and broad-spectrum antibiotics. Abdominal X-ray (B) does not accurately detect the presence of cholelithiasis and is not useful in the workup of cholecystitis. While gallstones can be seen on CT abdomen (C), approximately 20% of stones will be missed because they appear to be the same density as surrounding tissue. Therefore, CT abdomen is not the best test for visualizing gallstones. MRI abdomen (D) is not the best or most practical test to detect gallstones and is not routinely used in the workup of cholecystitis.

What is Gerstmann syndrome?

Combination of acalculia, agraphia, finger agnosia, and right-left disorientation seen in left posterior cerebral artery stroke.

What is the name for a unilateral adrenal adenoma?

Conn syndrome

A 45-year-old Native American woman presents to the emergency department with fever, jaundice, and right upper quadrant pain for six hours. Her pain is constant and she denies any nausea or vomiting. An abdominal ultrasound reveals the presence of a common bile duct stone with biliary duct dilatation but no other abnormal findings. Which of the following is the most likely diagnosis? A. Acute cholangitis B. Acute cholecystitis C. Acute viral hepatitis D. Biliary colic

Correct Answer: (A) Acute cholangitis Explanation: Acute cholangitis is a bacterial infection of the biliary tree due to an obstruction in the area, most commonly by a gallstone. Obstruction can also be due to a neoplasm or stricture. When the biliary tract is obstructed, intraluminal pressure is elevated, leading to an optimal environment for the bile (which is normally sterile) to become infected. Cholangitis will not typically develop without an obstruction. The infection can ascend from the tree into the hepatic ducts, hepatic veins, and perihepatic lymphatics leading to bacteremia. The most common organisms isolated in these cases ranging from most common to least common are Escherichia coli, Klebsiella species, Enterococcus species, Streptococcus species, Enterobacter sepsis, and Pseudomonas aeruginosa. There may be polymicrobial infection found in bile and blood cultures. Primary sclerosing cholangitis is a type of cholangitis that is a chronic liver disease associated with inflammation and fibrosis of the biliary tree and hepatic ducts. Choledocholithiasis (gallstones in the common bile duct) is the most common cause of acute cholangitis. Other less common causes are obstructive tumors (e.g., pancreatic cancer, cholangiocarcinoma), strictures, stenosis, endoscopic manipulation, sclerosing cholangitis, and after endoscopic retrograde cholangiopancreatography. Other risk factors include recent cholecystectomy, history of cholangitis, endoscopic manipulation of the biliary tree, and history of human immunodeficiency virus or acquired immunodeficiency syndrome. Patients with cholangitis will have a spectrum of presentations, ranging from mild symptoms to fulminant sepsis. Classic findings associated with this condition are right upper quadrant pain, fever, and jaundice (Charcot triad). If there are mental status changes and sepsis involved, these are grouped together as a Reynolds pentad. Other symptoms include chills, abdominal pain, pruritus, acholic stools, and malaise. Physical exam findings include fever, right upper quadrant tenderness, hepatomegaly, jaundice, hypotension, tachycardia, and rarely, peritonitis. Ultrasonography and computed tomography scanning are first-line imaging modalities. Transabdominal ultrasound is very sensitive and specific for examining the gallbladder and assessing bile duct dilatation and is very advantageous due to ease of use and lack of radiation. Computed tomography shows a more complete picture of the biliary tree and the surrounding structures and is able to detect radiolucent stones, as well as other complications of cholangitis (e.g., tumors, liver abscesses). Endoscopic retrograde cholangiography, magnetic resonance cholangiopancreatography, and endoscopic ultrasonography are often used for diagnostic and therapeutic purposes. Endoscopic retrograde cholangiography is considered the criterion standard for imaging the biliary tree and is reserved for patients who require therapeutic intervention. Laboratory studies such as complete blood count (which will reveal leukocytosis), liver function tests (which will reveal hyperbilirubinemia and elevated alkaline phosphatase), and blood cultures should be performed. Treatment depends on the severity of the patient's symptoms. Unstable patients should be closely monitored and stabilized with oxygen and intravenous fluids. Parenteral broad-spectrum antibiotics should be started empirically after blood cultures of drawn. In stable patients, medical treatment with parenteral antibiotics may be all that is required, with elective surgery being an option in the future. For those who do not respond to medical therapy or those who are severely ill, immediate emergency decompression via endoscopic retrograde cholangiopancreatography, sphincterotomy, or percutaneous drainage of the biliary tree may be required. Complications of cholangitis include liver failure, hepatic abscesses, sepsis, and acute renal failure. Acute cholecystitis (B) is not the best answer because the patient does not have any symptoms of nausea or vomiting, and her ultrasound does not mention abnormal gallbladder findings (e.g., thickened gallbladder wall, enlarged gallbladder). Patients with acute viral hepatitis (C) typically present with symptoms resembling acute gastroenteritis and may have pruritus, malaise, and fatigue with changes in the urine and stools. It is not caused by gallstones. Biliary colic (D) is incorrect because this patient's pain is constant, and does not wax and wane. Biliary colic pain usually lasts from one to several hours and often occurs after a heavy meal.

Which cranial nerve is the most vulnerable to the effects of increased intracranial pressure?

Cranial nerve VI (abducens nerve) due to its long course in the subarachnoid space.

A 14-year-old girl presents to the emergency room, refusing to give a history of her present illness. She is accompanied by her mother, who states that her daughter "has genital warts and needs the vaccine." Which of the following best represents appropriate health counseling for this patient? A. The human papillomavirus vaccine causes remission of human papillomavirus lesions. B. The human papillomavirus vaccine is contraindicated for immunocompromised patients. C. The human papillomavirus vaccine is recommended for girls only, ages 15 years and older. D. The human papillomavirus vaccine prevents initial human papillomavirus infection.

Correct Answer: (D) The human papillomavirus vaccine prevents initial human papillomavirus infection. Explanation: The human papillomavirus exists in over 200 strains and is responsible for cutaneous, anogenital, and oral lesions in human hosts. The virus is transmitted via physical contact with an infected individual, and most infections are transient, causing either no symptoms or flesh-colored, nonpruritic, verrucous papules on the infected areas. Certain strains of human papillomavirus are responsible for cytologic changes and cancer formation. Human papillomavirus types 16, 18, 31, 33, 35, 45, 52, and 58 are the most common types associated with the development of cervical cancer. Types 16 and 18 are associated with the development of anal cancer and penile cancer, and type 16 is associated with the development of squamous cell carcinoma of the oropharynx. The virus can be transmitted from mother to infant during childbirth, and types 6 and 11 can lead to recurrent respiratory papillomatosis in a child via maternal transmission. Treatment of human papillomavirus is focused on prevention, as there are no oral medications currently available that can cause eradication of the virus. Cutaneous lesions are often treated with topical cytotoxic agents for their individual removal. The human papillomavirus vaccine available in the United States targets types 6, 11, 16, 18, 31, 33, 45, 52, and 58 and is recommended for the prevention of human papillomavirus infection. The vaccine is ideally administered to girls and boys prior to sexual debut, ages 11 to 12 years, in a two-dose schedule. Dose two is given 6 to 12 months after the original dose. The vaccine may be given as early as 9 years and is not recommended after 26 years of age due to the likelihood of prior exposure. The human papillomavirus vaccine does not cause the remission of human papillomavirus lesions (A) or related disease. The girl in this vignette may benefit from vaccination, as it would provide protection against strains she has not yet been exposed to, but vaccination would not cause remission of lesions that have already appeared. The human papillomavirus vaccine is not a live virus and is recommended for immunocompromised individuals (B). Immunocompromised patients should be administered a three-dose regimen at intervals of zero, one or two, and six months. The human papillomavirus vaccine is recommended for girls (C) and boys, ages 11 or 12. Vaccination of boys serves both to prevent transmission of virus types that cause cervical cancer and also to prevent penile, anal, and oropharyngeal cancer in both sexes. The best time to vaccinate is prior to sexual debut. Currently, the vaccine is recommended for children as young as 9 years old.

A 32-year-old multiparous woman presents to the emergency department with vaginal bleeding after giving birth one hour earlier at home. She describes a prolonged labor and brings the intact placenta with her. On physical exam, she is mildly hypotensive and tachycardic, with a boggy uterus on palpation. Copious active uterine bleeding is noted, with total blood loss since arrival estimated at 500 mL. Bimanual uterine compression, oxygen administration, and an oxytocin drip with lactated Ringer solution are initiated while blood is drawn for labs, including blood typing. Which of the following is the best next step in management of this emergency? A. Emergency hysterectomy B. Exploratory laparotomy C. Intrauterine balloon tamponade D. Tranexamic acid 1 g IV

Correct Answer: (D) Tranexamic acid 1 g IV Explanation: Postpartum hemorrhage is defined as more than 500 mL of blood loss in the first 24 hours after vaginal childbirth or more than 1,000 mL of blood loss in the first 24 hours after cesarean section. It is a major cause of maternal mortality worldwide, especially in the developing world. Postpartum hemorrhage may result from uterine atony, uterine rupture, retained placenta, cervical or vaginal tears, hematomas, or coagulopathies. Those at increased risk for the development of postpartum hemorrhage include women with retained products of conception, abnormally adherent placenta, prolonged first or second stage of labor, lacerations, instrumental delivery, large infant size, intrauterine fetal demise, hypertensive disorders, and those who require induction of labor. Signs and symptoms of postpartum hemorrhage include brisk uterine or vaginal bleeding after delivery and a boggy uterus which does not become firm after uterine massage. Hypotension, tachycardia, pallor, hypothermia, and loss of consciousness or death may occur, representing various stages of hypovolemia or shock. Initial evaluation of a patient with postpartum hemorrhage includes careful assessment of the amount of blood loss, attention to vital signs, volume repletion through intravenous fluids, and rapid physical examination for any obvious cause of bleeding (e.g., hematoma, laceration, or retained placenta). Uterine massage and bimanual compression of the uterus should be performed if atony is suspected. Also, 15 units of oxytocin should be infused in 250 mL of lactated Ringer solution. If the uterus is still boggy and bleeding continues, tranexamic acid 1 g (an antifibrinolytic agent) is administered intravenously over 10 minutes. Upon further bleeding, the patient can be given other uterotonics such as methergine, carboprost, or misoprostol. During the entire process, oxygen should be administered, at least one large- bore intravenous access should be attained, blood should be drawn for blood typing, and the patient should be kept warm. Lacerations and hematomas should be managed. An intrauterine balloon catheter may be used if atony and hemorrhage persist after administration of uterotonic agents. If blood loss exceeds 1,000 mL, transfusion should be given. If minimally invasive procedures fail to stop the hemorrhage, an emergency laparotomy, uterine artery embolization, uterine artery ligation, or a B-Lynch uterine compression suture may be necessary. The final step, when all else fails, is a hysterectomy. Prevention of postpartum hemorrhage involves avoidance of prolonged labor through use of oxytocin when necessary and avoidance of lacerations and hematomas through decreased manipulation and instrumentation during labor. Emergency hysterectomy (A) should be reserved as the last option for treatment of postpartum hemorrhage, as preservation of fertility is often a primary patient concern. Avoidance of surgical procedures, such as hysterectomy, is also optimal in a setting of bleeding, hemorrhage, and potential coagulopathy. Exploratory laparotomy (B) is used much earlier in an algorithm of postpartum hemorrhage when the patient has received a cesarean section, as the abdomen is already open. In a vaginal delivery, exploratory laparotomy should be avoided unless other, minimally invasive measures have failed. Intrauterine balloon tamponade (C) is a technique which uses a catheter-type balloon to provide physical compression of intrauterine arteries in an effort to staunch blood flow. The device is very effective but should be used if bleeding continues after manual compression, oxytocin, and tranexamic acid have been administered. The device has two lumens, one for inflating the balloon and another for drainage of uterine blood so that continued hemorrhaging can be identified and quantified.

Bulimia Nervosa

DSM-5: recurrent episodes of binge eating followed by inappropriate compensatory behavior via self-induced vomiting, laxative misuse, excessive exercise, or caloric restriction (occurring at least once per week for 3 months) Sense of lack of control during eating episodes Self-evaluation is unduly influenced by body shape or weight PE: body weight usually within or above normal range, dental erosions, parotid gland swelling, callused knuckles Tx options: cognitive behavioral therapy, fluoxetine or other SSRIs, or combined CBT/pharmacotherapy

Multiple Sclerosis

Demyelinating CNS lesions separated in space and time Risk factors: female sex, age < 30, birth at northern latitude (possibly linked to vitamin D deficiency) Sx: pain with eye movement, monocular vision loss, diplopia with lateral gaze, sensory abnormalities PE: afferent pupillary defect (Marcus Gunn pupil), spinal electric shock sensation with neck flexion (Lhermitte sign), impaired bilateral adduction (bilateral internuclear ophthalmoplegia) Dx: MRI: periventricular white matter lesions CSF: oligoclonal IgG bands, WBC pleocytosis Tx: Acute exacerbation: methylprednisolone Disease modifying: natalizumab, ocrelizumab, glatiramer acetate, interferon beta-1a Note: Natalizumab associated with PML, test for JC virus antibodies

Why is a lumbar puncture contraindicated in patients with subdural hematoma?

Due to the risk of herniation

What is the name of the eye pairs of a brown recluse spider?

Dyad

Which viruses are known to provoke aplastic anemia?

Epstein-Barr virus, seronegative hepatitis, herpes virus, and human immunodeficiency virus.

What is the most common cause of pulmonary embolism in Caucasian populations?

Factor V Leiden deficiency

True or false: women who received a complete regimen of the human papillomavirus vaccine before sexual debut have no need of annual screening pap smears?

False, the vaccine does not protect against all strains of human papillomavirus that may cause cervical cancer.

What are the most common causes of cirrhosis in the United States?

Hepatitis C, alcoholic liver disease, and nonalcoholic fatty liver disease

What are the two most common viral causes of corneal ulcerations?

Herpes simplex and Varicella zoster

Epidural Hematoma

History of a head injury with a loss of consciousness followed by a lucid interval CT will show a biconvex opacity Most common artery ruptured is the middle meningeal artery Treatment is emergent evacuation

Hyphema

History of blunt or penetrating trauma Blurry vision PE will show unequal pupils, injected conjunctiva or sclera, and blood in anterior chamber Treatment is with eye protection, limitation of activity, and head elevation of 30-45 degrees

Hepatic Encephalopathy

History of infection, dehydration, medications, GI bleed, hypoxia, hypoglycemia Presents with confusion PE will show scleral icterus and asterixis Most commonly caused by toxin accumulation due to liver dysfunction Treatment is lactulose, rifaximin CSF glutamine correlates with severity

What underlying condition is associated with torsade de pointes finding on ECG?

Hypokalemia, hypomagnesemia, and certain drugs that cause QT prolongation (e.g., class IA and class III antidysrhythmics, azithromycin, levofloxacin).

What is HELLP syndrome?

It is a group of symptoms that occur in pregnant women who have hemolysis, elevated liver enzymes, and low platelet count.

Hepatitis B

New CDC guidelines: In addition to groups for whom HepB vaccination is already recommended, the Advisory Committee on Immunization Practices recommends that all adults aged 19-59 years should receive HepB vaccines. Acute or chronic infection with DNA virus Transmission: perinatal, blood, sex Chronic infection: cirrosis (20%), HCC (10%) Surveillance with abdominal imaging q6 months Dx: Acute hepatitis B: HBsAg, Anti-HBc IgM, HBeAg,+ HBV DNA Resolved infection: Anti-HBs, Anti-HBc IgG, +/− HBeAb Vaccinated: Anti-HBs Tx: Acute: supportive care, antivirals if acute liver failure Chronic: Tx if active hepatic inflammation or cirrhosis Entecavir, tenofovir, lamivudine, interferon

What kind of bacteria are found in the genus Shigella?

Nonmotile, facultatively anaerobic, gram-negative rods.

What groups of people have the highest prevalence of gallstones?

Northern Europeans, Hispanics, Native Americans, and Pima Indians

What is the most common cause of brain abscess in individuals who have lived outside the United States?

Parasites

Subarachnoid Hemorrhage

Patient presents with abrupt onset of "worst headache of life" or thunderclap headache Diagnosis is made by noncontrast CT scan, blood will appear white on the CT If CT negative and suspicion high, lumbar puncture. Most commonly caused by a ruptured aneurysm Hunt & Hess classifies severity of subarachnoid hemorrhage to predict mortality Treatment is supportive and nimodipine (decreases vasospasm)

Papilledema

Patient presents with acute onset of intermittent headaches and blurred vision PE will show optic disk swelling Focus on finding intracranial pathology Most commonly caused secondary to increased intracranial pressure Treatment is the treatment of the underlying disorder

Tinea Versicolor

Patient presents with hypopigmented areas that do not tan PE will show scaly patches on the chest and trunk Diagnosis is made by KOH preparation of skin scraping Most commonly caused by Malassezia species (e.g., M. furfur) Treatment is topical selenium sulfide 2.5% or topical ketoconazole

Placental Abruption

Patient will be in third trimester History of hypertension, trauma, or cocaine use Painful vaginal bleeding Labs will show hypofibrinogenemia Tx: fetal monitoring, hemodynamic stabilization, delivery

What is the best noninvasive study to evaluate for perianal fistulas?

Pelvic MRI

What class of medications is indicated to additionally lower low-density lipoprotein levels in patients with maximally tolerated statin therapy?

Proprotein convertase subtilisin-kexin type 9 (PCSK9) inhibitors, such as evolocumab and alirocumab.

Second-Degree Heart Block (Mobitz I, Wenckebach)

Rhythm will be irregular PR interval will be progressively lengthening Notable feature: progressive lengthening of the PR interval until QRS complex fails to appear after a P wave (dropped beat)

Subdural Hematoma

Risk factors: traumatic head injury, advancing age, anticoagulant use, coagulopathy, thrombocytopenia Caused by tearing of the bridging veins between arachnoid and dura Sx: acute or subacute neuro sx, headache, mental status changes, seizures, or focal deficits Dx: crescent-shaped hematoma on noncontrast CT Management includes neurosurgical consultation, blood pressure control, reversal of anticoagulation

What is the role of melatonin in hepatic encephalopathy?

Sleep disturbance is a common symptom of hepatic encephalopathy and may be corrected with use of melatonin, but studies have shown no significant clinical improvement with its use.

Open Fracture

Surgical evaluation ASAP Administer antibiotics, gram-positive coverage Cover for gram-negative and MRSA if needed Update tetanus Consider bone biopsy or cultures in OR

Olecranon Bursitis

Swelling, pain ↓ active ROM Full passive ROM NSAIDs

Cholecystitis

Sx: colicky, steadily increasing RUQ or epigastric pain after eating fatty foods PE: Murphy sign, Boas sign (hyperaesthesia, increased or altered sensitivity, below the right scapula) Diagnosis Initial: U/S Gold standard: HIDA Most commonly caused by obstruction by a gallstone Treatment is cholecystectomy

Pulmonary Embolism

Sx: dyspnea, pleuritic chest pain, cough, syncope PE: tachypnea, tachycardia, possible signs of DVT (calf pain or swelling) ECG: sinus tachycardia, nonspecific ST segment and T wave changes, RV strain, S1Q3T3 (classic finding) CXR: nonspecific abnormalities, Hampton hump (pleural-based wedge infarct), Westermark sign (vascular cutoff sign) Dx: CT pulmonary angiography most preferred, VQ scan alternative Tx: -Anticoagulation: heparin, LMWH, warfarin, novel oral anticoagulants (NOAC) -Thrombolytics, embolectomy in hemodynamically unstable patients Comment: in low clinical suspicion: negative D-dimer excludes PE

Nephrolithiasis

Sx: flank pain radiating to groin PE: patient won't lie still, hematuria Diagnosis: noncontrast helical CT, most common location is the ureterovesical junction (UVJ) Most commonly caused by calcium oxalate Struvite: staghorn calculi, urease-producing bacteria Uric acid: radiolucent on X-ray, gout Cystine: children with metabolic diseases Treatment: < 5 mm: likely to pass spontaneously > 5 mm: medical expulsive therapy (tamsulosin), urology consultation in certain cases > 10 mm: urology consultation, shock wave lithotripsy, ureteroscopy

Which patients are candidates for antibiotic therapy prior to dental procedures?

Those with prosthetic heart valves, previous history of infectious endocarditis, some forms of congenital heart disease (e.g., cyanotic congenital heart defects), and cardiac transplant patients with valvular disorders.

Which fractures have the highest risk of nonunion?

Tibia fractures have the highest nonunion rate of any site, and open fractures have more than 20 times the risk of nonunion than closed fractures.

What is the etiology of pityriasis rosea?

Viral

What is the recommended treatment duration of bulimia nervosa?

6 to 12 months after response or remission

A 40-year-old woman presents after she sustained a painful bite by an unknown insect three days ago. She was cleaning out her attic when she felt a painful bite on her hand. The bite was initially described as a red plaque that has progressed to a dusky red eschar. Physical examination reveals a necrotic ulcer. Which of the following spiders most likely bit her? A. Black widow spider B. Brown recluse spider C. Funnel web spider D. Yellow sac spider

Correct Answer: (B) Brown recluse spider Explanation: Brown recluse spiders belong to the genus Loxosceles and are also known as Loxosceles reclusa. Identification of a brown recluse spider on the basis of body morphology is less reliable than counting the eyes. Brown recluse spiders have six eyes, with a pair in front and a pair on either side with a gap between the pairs. Other spiders have eight eyes. Brown recluse spiders are brown with a violin pattern on the anterior cephalothorax and are distributed in the Central Midwestern United States. They are typically found in dark, quiet, and undisturbed areas, such as basements, attics, behind bookshelves, and under rocks. Clinical manifestations of a brown recluse spider bite or loxoscelism include a painful bite reported by patients that is followed by local signs or systemic effects or both. Local signs may be identified as two puncture marks surrounded by an area of erythema. The bite is usually a painful red plaque or papule. Pain typically intensifies after two to eight hours and may become severe. Over several days, the red plaque or papule develops a dusky red or blue color in the center of the lesion forming an eschar. As the eschar breaks down, a necrotic ulcer typically develops. Systemic effects include malaise, fever, myalgias, nausea, or vomiting. Rarely, systemic effects may include angioedema, acute hemolytic anemia, rhabdomyolysis, myonecrosis, coma, and even death. Diagnosis is made clinically based on the history of feeling a bite and the presence of a necrotic wound. Definitive diagnosis is made only when a patient has a consistent skin lesion that was inflicted by an observed spider that was then recovered and properly identified by an entomologist. Laboratory studies are usually unnecessary initially when only local signs are present but may be needed in the later stages as systemic effects develop. Acute hemolysis, rhabdomyolysis, or acute renal injury may be seen on lab study. Treatment may be achieved by extensive wound cleaning and analgesia with nonsteroidal anti-inflammatory drugs or opioids as needed. Excision/debridement of the wound in the necrotic stage is not recommended but once the wound is stable, this may help expedite wound healing. Black widow spider (A) typically causes unremarkable local lesions occasionally accompanied by systemic reaction, including progressive muscle pain, nausea, vomiting, and diaphoresis. Funnel web spider (C) is indigenous to Eastern Australia and causes dramatic systemic reaction that mimics organophosphate poisoning, including salivation, diaphoresis, muscle spasms, tachycardia, hypertension, muscle fasciculations, and paresthesia. Individuals with a yellow sac spider (D) bite complain of a painful bite similar to a bee sting that is typically not associated with skin necrosis. The bite is typically accompanied by redness, swelling, and pruritus.

A 22-year-old woman with a history of diabetes mellitus presents with abdominal pain and vomiting. She has not been taking her insulin because she has been eating less the past couple days. Her initial serum glucose is 270 mg/dL. If the patient is in diabetic ketoacidosis, which of the following additional findings is most likely? A. Arterial pH of 7.4 B. Decreased serum bicarbonate C. Hypernatremia D. Hypokalemia

Correct Answer: (B) Decreased serum bicarbonate Explanation: Diabetic ketoacidosis is one of the most serious and acute complications of diabetes mellitus. It is characterized by hyperglycemia and ketoacidosis. Diabetic ketoacidosis is more common in younger patients with type 1 diabetes mellitus. Hyperglycemia is caused by impaired glucose utilization, increased gluconeogenesis, and increased glycogenolysis. Gluconeogenesis results from the breakdown of fat and muscles and is promoted by insulin deficiency. Ketoacidosis results from lipolysis and the synthesis of ketones from free fatty acids. Diabetic ketoacidosis is typically triggered by precipitating events, such as infection (pneumonia and urinary tract infections are most common), inadequate insulin, or noncompliance with insulin. It may be the initial presentation of type 1 diabetes mellitus. Diabetic ketoacidosis presents with symptoms related to hyperglycemia, abdominal symptoms, dehydration, and symptoms of the precipitating infection. Hyperglycemic symptoms include polyuria, polydipsia, and weight loss. Abdominal symptoms, such as abdominal pain, nausea, and vomiting, are more common in severe diabetic ketoacidosis, caused by delayed gastric emptying and ileus, and are due to acidemia. Neurologic manifestations, such as decreased awareness and stupor, may be seen in severe diabetic ketoacidosis but are more common in hyperosmolar hyperglycemic state. Hyperventilation with fruity breath is common in diabetic ketoacidosis. Dehydration manifests as tachycardia, orthostatic hypotension, dry mucous membranes, and decreased skin turgor. The diagnosis of diabetic ketoacidosis is confirmed with laboratory testing. The most common laboratory findings of diabetic ketoacidosis are hyperglycemia, hyperosmolality, elevated anion gap, decreased serum bicarbonate, urine ketones, and serum ketonemia. Beta-hydroxybutyrate is often measured to check for elevation in serum ketones. The arterial pH from an arterial blood gas helps to decipher the severity of diabetic ketoacidosis. In general, an arterial pH of 7.25-7.30 indicates mild diabetic ketoacidosis, whereas an arterial pH less than 7.0 indicates severe diabetic ketoacidosis. Patients with diabetic ketoacidosis are typically managed in an intensive care unit. The treatment involves intravenous fluids to help with dehydration and hyperosmolality, potassium, insulin, and sometimes sodium bicarbonate. Patients with diabetic ketoacidosis typically have depleted total body potassium. However, due to transcellular shift, the initial serum potassium may be normal or elevated. The administration of insulin moves potassium back into the cells, which can cause sudden development of hypokalemia. Therefore, patients with an initial potassium between 3.3-5.3 mEq/L should be given intravenous potassium chloride along with intravenous fluids and insulin. Patients with diabetic ketoacidosis and an initial potassium below 3.3 mEq/L should be given intravenous fluids and potassium prior to the administration of insulin. Insulin is given intravenously in moderate-severe diabetic ketoacidosis. An initial bolus may be given, and then a continuous insulin infusion (insulin drip) follows. Sodium bicarbonate is recommended in cases with an arterial pH below 6.9. The serum glucose should be monitored hourly. The ketoacidosis can be monitored every two to four hours with venous blood gas, chemistry panels, or serum ketone measurements. An arterial pH of 7.4 (A) is normal. The normal arterial pH is between 7.35-7.45. Patients with diabetic ketoacidosis typically have an acidic pH. Hypokalemia (D) is uncommon on the initial serum potassium. Most patients with diabetic ketoacidosis have a potassium deficit of 300-600 mEq/L. However, the insulin deficiency causes potassium to move from the intracellular space to the extracellular space, which causes the serum potassium to usually be normal in most patients and high in one-third of patients. Hypokalemia is only seen in 5% of patients with diabetic ketoacidosis. However, the administration of insulin can abruptly lower potassium. Therefore, potassium supplementation as insulin is administered is imperative in patients with diabetic ketoacidosis. Hypernatremia (C) is uncommon in diabetic ketoacidosis. Most patients have mild hyponatremia primarily due to the increase in plasma osmolality from hyperglycemia, which pulls water out of the cells.

A 46-year-old man presents to the clinic for a routine physical. He has a medical history of coronary artery disease, hypertension, hyperlipidemia, and a previous myocardial infarction. His vitals reveal HR 75 bpm, RR 15/min, BP 148/87 mm Hg, T 98.7°F, and SpO2 99% on room air. His most recent lipid panel reveals low-density lipoprotein cholesterol of 92 mg/dL, high-density lipoprotein cholesterol of 41 mg/dL, triglyceride level of 394 mg/dL, and total cholesterol of 212 mmol/L. Which of the following should be the primary goal in reducing this patient's risk for acute coronary syndrome? A. Decreasing blood pressure using amlodipine B. Decreasing LDL levels using atorvastatin C. Decreasing triglyceride levels using gemfibrozil D. Increasing HDL levels using niacin

Correct Answer: (B) Decreasing LDL levels using atorvastatin Explanation: Coronary heart disease is a pathologic process that affects the coronary circulation and results in acute coronary events, such as angina, myocardial infarction, and cerebrovascular accident. It is the number one cause of death worldwide and is responsible for approximately one in five deaths in the United States each year. Risk factors for developing coronary artery disease include family history of premature coronary heart disease, smoking, hypertension, high-density lipoprotein levels below 40 mg/dL, and age ≥ 45 in men or ≥ 55 in women. Additionally, patients with diabetes mellitus and those who consume too much alcohol and too few fruits and vegetables are at an increased risk for developing coronary heart disease. High-density lipoprotein levels ≥ 60 mg/dL have been demonstrated to be cardioprotective. Smoking remains the most important modifiable risk factor, with the risk of coronary heart disease decreasing by 50% one year after smoking cessation. Risk factor reduction is an important component of reducing the morbidity and mortality in susceptible patients. In clinical trials, only the reduction of elevated low-density lipoprotein cholesterol levels has demonstrated clinical benefit. Medical management of elevated serum triglycerides or low levels of high-density lipoprotein cholesterol has not shown significant benefit in the primary prevention of coronary heart disease. Studies have demonstrated that hydroxymethylglutaryl-coenzyme A (HMG-CoA) reductase inhibitors (also called statins) are effective in preventing death, coronary events, and stroke and should be administered to all patients at risk of these events regardless of cholesterol levels. Although an overall target low-density lipoprotein cholesterol level is not recommended, many guidelines for statin therapy recommend that those with prior cardiovascular events maintain low-density lipoprotein cholesterol levels below 70 mg/dL. High-intensity statins include atorvastatin and rosuvastatin in doses of 40-80 mg or 20-40 mg, respectively. This group tends to lower low-density lipoprotein cholesterol levels by over 50%. Low-intensity statin therapy lowers low-density lipoprotein levels by less than 30% and consists of low-dose simvastatin, pravastatin, lovastatin, fluvastatin, and pitavastatin. Moderate-intensity statin therapy includes those agents used for high-intensity therapy at lower doses as well as higher doses of low-intensity medications. Patients at risk for coronary heart disease benefit most from high- or moderate-intensity statin therapy. Myalgia is the primary complaint that causes statin intolerance and can be exacerbated in patients with hypothyroidism. Other side effects include elevated aminotransferases, proteinuria, increased risk of diabetes mellitus, and drug-induced lupus. Statins are a pregnancy category X drug and are contraindicated in pregnant and nursing women. Decreasing blood pressure using amlodipine (A), which is a calcium channel blocker, has not been shown to decrease the risk of mortality or morbidity in patients with coronary heart disease. The Heart Outcomes Prevention Evaluation (HOPE) trial and European trial on reduction of cardiac events with perindopril in stable coronary artery disease (EUROPA) have demonstrated benefit with the use of an angiotensin-converting enzyme inhibitor (e.g., enalapril) in patients with vascular disease, even in the absence of heart failure or left ventricular dysfunction. Decreasing triglyceride levels using gemfibrozil (C) is not the best approach to reducing the risk of a cardiovascular event in this patient. Unless triglyceride levels exceed 500 mg/dL despite dietary intervention, the use of medications to lower this value has not been proven to substantially affect the course of cardiovascular disease. Increasing high-density lipoprotein levels using niacin (D) has not been shown to reduce the cardiovascular risk in patients with coronary heart disease. Additionally, the best treatment approach for patients with low levels of high-density lipoprotein cholesterol is exercise, weight loss, smoking cessation, and dietary modification (e.g., substitution of monounsaturated for saturated fatty acids).

A 20-year-old man presents to the emergency department complaining of a foreign body sensation in the right eye accompanied by blurred vision and redness of his eyelid. He wears soft contacts and says he has been sleeping in them most of last week because of final exams. On physical exam, a lesion with a white, hazy base is visualized along with purulent discharge from the eye and erythema of the eyelid. Which of the following is the best treatment for this patient? A. Place a patch over the eye B. Prescribe topical amphotericin B C. Prescribe topical ciprofloxacin D. Prescribe topical cyclopentolate

Correct Answer: (C) Prescribe topical ciprofloxacin Explanation: This patient is presenting with a corneal ulceration, which should be treated with topical ciprofloxacin. Corneal ulcerations occur as a result of disruption of the epithelial barrier. Trauma, exposure keratitis (commonly seen with Bell palsy), or direct microbial invasion can lead to corneal ulceration. Contact lens wearers are especially prone to this condition, and it is more commonly seen in soft contact lens wearers who have either been sleeping in their contacts or who have delayed changing to a new pair. In noncontact lens-related corneal ulcerations, the most common organisms are Streptococcus pneumoniae and Staphylococcus aureus. Pseudomonas aeruginosa is the most common organism in contact lens wearers. Viral and fungal causes are less common than bacterial organisms, however, there has been a rise in the number of viral and fungal infections due to increased use of immunosuppressant therapy to treat other disorders. Patients commonly complain of eyelid erythema and edema, purulent eye discharge, foreign body sensation, photophobia, and blurred vision. While vision is typically preserved, ulcerations occurring in the central line of vision or affecting the uveal tract will cause a decrease in vision as assessed by visual acuity testing. Physical exam shows an ulceration of the corneal surface that appears white and hazy. The diagnosis is made clinically. An ophthalmologist typically cultures the ulceration. The best treatment is a topical fluoroquinolone applied to the affected eye every hour. Patients should be seen by an ophthalmologist within 12-24 hours. Complications include corneal scarring, perforation, glaucoma, and cataract formation. Placing a patch over the eye (A) is not recommended. In patients with Pseudomonas aeruginosa, patching the eye accelerates ulceration and can lead to melting and perforating the cornea. Prescribe amphotericin B (B) is not recommended. Since the most common causes of corneal ulceration are bacterial, treating empirically with an antifungal is incorrect. Prescribing topical cyclopentolate (D), a topical cycloplegic, can help manage pain associated with corneal ulcerations particularly if there is accompanying uveitis. It does not directly treat the infection causing the ulceration.

A 76-year-old woman with hyperlipidemia presents to the emergency department for sudden onset of visual disturbances, which resulted in a motor vehicle collision. She is not on any antiplatelet medication. There is no hemiparesis or sensory disturbance on her exam. A left homonymous hemianopia is noted. Noncontrast CT scan of the brain is performed, which demonstrates evidence of an acute infarct. Based on her presenting symptoms, which artery is likely affected? A. Left middle cerebral artery B. Left posterior cerebral artery C. Right posterior cerebral artery D. Right posterior inferior cerebellar artery

Correct Answer: (C) Right posterior cerebral artery Explanation: Infarcts involving the right posterior cerebral artery often result in a left homonymous hemianopia. Depending on where in the occipital lobe the infarct occurs, the macula can be spared. If the full territory of the posterior cerebral artery is involved, a patient can also develop visual neglect. Patients can develop paresthesia and numbness if the lateral thalamus is involved. Other symptoms demonstrated in posterior cerebral artery strokes are alexia without agraphia, anomic aphasia, sensory aphasia, defective acquisition of new memories, prosopagnosia, and dreams without visual imagery. A stroke, or cerebrovascular accident, can result from an ischemic (thrombus or embolus) or hemorrhagic etiology. Risk factors include hypertension, hyperlipidemia, tobacco use, diabetes, underlying cardiac disease, cocaine use, hypercoagulable disorder, and family history of stroke. The presenting symptom is dependent on the area of the brain and artery affected. The common symptoms attributed to stroke are facial droop, slurred speech, and contralateral weakness. Noncontrast CT scan of the head must be performed emergently to evaluate whether the stroke is hemorrhagic or ischemic in nature. Magnetic resonance imaging of the brain should be considered in the event the CT scan is normal, but high suspicion for stroke remains. Angiography may be performed to evaluate for evidence of hemodynamically significant stenosis or vascular occlusion. Recombinant tissue plasminogen activator (r-TPA) alteplase should be given in the event of an ischemic stroke presenting within four and a half hours, given that the patient does not meet thrombolytic exclusion criteria. Blood pressure control is imperative to ensure adequate perfusion to the brain. High-dose statin and antiplatelet therapy are needed for secondary stroke prevention. Anticoagulants should be considered in the appropriate patient populations if embolic etiology is a concern. Left middle cerebral artery (A) is incorrect. This would result in aphasia and right-sided hemiparesis. Left posterior cerebral artery (B) is incorrect. This would result in right-sided homonymous hemianopia. Right posterior inferior cerebellar artery (D) is incorrect. A stroke involving the cerebellum would result in ataxia and limb incoordination.

A 25-year-old man presents to the emergency department after he was thrown from a motorcycle when he was struck by a car. He was wearing a helmet and did not lose consciousness. He complains of right lower leg pain. On examination, he has a 5-centimeter wound over the anterior tibia. There is an obvious fracture of the midshaft of the tibia with bone ends visible in the wound and moderate muscle and skin damage but no large area of soft tissue loss. He is neurovascularly intact distally. Which of the following initial interventions has been shown to be most important in preventing complications of this injury? A. Administer IV antibiotics B. High-pressure irrigation with antibiotic solution C. Obtain wound cultures in the ER D. Perform surgical stabilization w/in 6 hours of injury

Correct Answer: (A) Administer IV antibiotics Explanation: Open fractures are defined as fractures that communicate with the external environment due to skin and soft tissue injury. The incidence of infection in open fractures is increased over closed fractures and correlates with the severity of the injury. Several classification systems have been used to describe open fractures. The Gustilo-Anderson classification is one of the most commonly used. Type I fractures are low-energy fractures with minimal soft-tissue damage and are typically described as a wound of 1-centimeter or less. Type II fractures involve moderate soft tissue damage, and wounds are between 1 and 10 centimeters with minimal contamination. Type III fractures are high-energy fractures with large wounds and extensive soft tissue damage with variable contamination. Farm injuries and fractures with associated vascular injuries are included in some subclassifications. Immediate administration of intravenous antibiotics is indicated, as the risk of infection has been shown to be increased when antibiotics are given more than three hours after injury. The choice of antibiotic is determined by the fracture type. A first-generation cephalosporin is recommended for Gustilo-Anderson Type I and II fractures, an aminoglycoside or third-generation cephalosporin is added for Type III fractures, and penicillin is added for gross contamination with soil. Initial treatment in the emergency department also involves immobilizing the extremity and tetanus prophylaxis as indicated by immunization status. Gross debris should be removed, and a sterile, saline-soaked dressing should be applied. Debridement and stabilization of the fracture are performed urgently, within 24 hours of injury, in the operating room. Repeat debridements may be necessary, but early closure or coverage of the wound is preferred. Most Type I and II fractures can be closed primarily, but Type III fractures may need delayed soft tissue reconstruction for wound coverage. High-pressure irrigation with antibiotic solution (B) is not indicated as high pressure can push debris further into the wound and antibiotics can be toxic to tissues. High volume, low-pressure irrigation with saline has been shown to be most effective. Obtaining wound cultures in the emergency department (C) is not useful for determining the choice of antibiotic or for predicting future infecting organisms and is not recommended. Performing surgical stabilization within six hours of injury (D) has not been shown to lower complications compared to within 24 hours of injury.

A 28-year-old woman presents to the emergency department complaining of fatigue, dyspnea, epistaxis, and increased bruising for the past week. Her past medical history is positive for seizure disorder, and she states her antiepileptic medications were changed last week. Physical exam reveals pallor, tachycardia, and widespread ecchymoses. No organomegaly is noted. Laboratory analyses reveal leukocytopenia, thrombocytopenia, and a low reticulocyte count. Hemoglobin is 7.2 g/dL, and the mean corpuscular volume is 110 fL. No abnormal cells are present on peripheral smear. Which of the following is the most likely diagnosis? A. Aplastic anemia B. Hemolytic anemia C. Macrocytic anemia D. Sickle cell anemia

Correct Answer: (A) Aplastic anemia Explanation: Aplastic anemia is defined as failure of the bone marrow resulting in pancytopenia. Injury to hematopoietic stem cells in the bone marrow results in decreased production of red blood cells, white blood cells, and platelets. Causes of aplastic anemia include autoimmune disease, viral infection, genetic disorders, and direct insult through chemicals, drugs, or irradiation. Men and women are equally affected, and aplastic anemia can occur at any age. Signs and symptoms of aplastic anemia include pallor, jaundice (if hemolysis is present), ecchymosis, bleeding, tachycardia, fatigue, peripheral edema (or other signs of heart failure), and increased incidence of bacterial or fungal infection. Patients may also present with a history of recent exposure to toxic substances, new drugs, or viral pathogens, or they may present with short stature, skin or nail changes, or other signs of a genetic disorder. Laboratory testing will reveal pancytopenia, which may vary from mild to severe, as well as a decreased reticulocyte count. Mean corpuscular volume may be normal or increased, and no abnormally shaped cells should be noted. Bone marrow biopsy is necessary for definitive diagnosis of aplastic anemia and will show decreased cellularity. Treatment of aplastic anemia involves recognizing and treating the underlying cause. Drugs that are known to cause aplastic anemia should be discontinued. This list includes cytotoxic agents, antibacterials, antiseizure medications, antithyroid drugs, nonsteroidal anti-inflammatory drugs, gold, and arsenic compounds. Overwhelming infection should be treated, and supportive care provided. In the absence of an identified, modifiable, causative insult, first-line therapy for severe aplastic anemia is with hematopoietic cell transplantation (depending on donor availability). If a donor is not available, immunosuppressive therapy may be life-saving. Hemolytic anemia (B) may present with pallor, dyspnea, fatigue, signs of heart failure, or jaundice. However, low platelet counts and low white blood cells, along with their concurrent signs and symptoms, would not be expected findings. Also, the reticulocyte count is most often elevated in hemolytic anemia, because the bone marrow is functioning well and working to replace hemolyzed red blood cells. By contrast, in aplastic anemia, the bone marrow has failed or is dysfunctional and the reticulocyte count is low. Macrocytic anemia (C) presents as low hemoglobin and increased mean corpuscular volume. This patient has macrocytic red blood cells with a low hemoglobin but also presents with leukocytopenia and thrombocytopenia, making aplastic anemia a more appropriate diagnosis for this patient. Sickle cell anemia (D) presents with normochromic, normocytic anemia, and an abnormal peripheral smear with sickle cells and target cells. Pancytopenia is not a finding in sickle cell anemia.

A 30-year-old woman who is 32 weeks pregnant presents to the emergency department with painful vaginal bleeding and uterine contractions that began abruptly about 30 minutes ago. The contractions are persistent, getting stronger, and do not have a consistent pattern. Physical exam shows marked abdominal tenderness and decreased fetal movement. Which of the following is a strong risk factor for this condition? A. Current tobacco use B. Gestational diabetes C. Obesity D. PCOS

Correct Answer: (A) Current tobacco use Explanation: Abruptio placentae, also known as placental abruption, is the premature separation of the placenta from the uterus. It should be considered in all patients who present with painful bleeding in the second half of their pregnancy, as it has a high fetal and maternal morbidity and mortality rate. Uterine hemorrhage leads to hematoma formation, causing compression of the uterus and compromising the blood supply to the fetus. The muscle wall in this area of the uterus can be weakened and can increase the risk of uterine rupture, which is a life-threatening emergency. Patients most commonly present with painful vaginal bleeding, but bleeding may not always be present. Other symptoms include contractions, abdominal tenderness, and decreased fetal movement indicating fetal distress. Patients who have a history of previous placental abruption are at the highest risk. Possible causes including trauma, such as assault, abuse, or motor vehicle collisions, should also be considered. Correctable risk factors include smoking and cocaine abuse, and cessation of these two activities may help to prevent future abruptions. Ultrasound of the uterus should be performed to exclude placenta previa before performing a pelvic examination, which can increase the risk of hemorrhage. Patients should be admitted to the hospital with possible delivery if abruptio placentae is suspected. Cesarean delivery is necessary to save the mother and fetus, with hysterectomy being required if hemorrhage cannot be controlled after delivery. Gestational diabetes (B) is not a risk factor for placental abruption but can cause complications like premature birth, stillbirth, and macrosomia. Obesity (C) is a risk factor for gestational diabetes, premature birth, stillbirth, and preeclampsia, but not placental abruption. Polycystic ovarian syndrome (D) can lead to increased risk of miscarriage, preeclampsia, gestational diabetes, and premature and stillbirth but is not a risk factor for placental abruption.Gestational diabetes (B) is not a risk factor for placental abruption but can cause complications like premature birth, stillbirth, and macrosomia. Obesity (C) is a risk factor for gestational diabetes, premature birth, stillbirth, and preeclampsia, but not placental abruption. Polycystic ovarian syndrome (D) can lead to increased risk of miscarriage, preeclampsia, gestational diabetes, and premature and stillbirth but is not a risk factor for placental abruption.

Which of the following represents appropriate health maintenance counseling for a patient with hepatic insufficiency and several previous episodes of hepatic encephalopathy? A. Daily oral probiotics B. Low-protein diet C. Two or three large meals daily with no snacking D. Zinc supplementation

Correct Answer: (A) Daily oral probiotics Explanation: Hepatic encephalopathy refers to neurologic changes as the result of hepatic insufficiency. Most patients with hepatic encephalopathy have increased levels of arterial ammonia (which is neurotoxic) due to decreased clearance of ammonia via hepatic enzymes. Other causes of hepatic encephalopathy include electrolyte imbalance, oxidative stress, and elevated 3-nitrotyrosine levels. All patients with hepatic encephalopathy have underlying liver disease (usually severe), and precipitating factors for the development of hepatic encephalopathy include infection, drugs, dehydration, portosystemic shunting, constipation, gastrointestinal bleeding, and excess dietary protein intake. Signs and symptoms of hepatic encephalopathy include flapping of the hands on forced dorsiflexion (asterixis), slurred speech, ataxia, memory deficits, slowed motor reaction time, increased deep tendon reflexes, agitation, depression, and sleep disturbances. Diagnosis of hepatic encephalopathy is with laboratory testing, which will demonstrate hepatic failure and may demonstrate hyperammonemia, as well as computed tomography of the head, which will rule out other neurologic causes of encephalopathy and will demonstrate focal or diffuse cerebral edema. Electroencephalogram can be employed in cases of mild hepatic encephalopathy where the diagnosis is unclear. Treatment of hepatic encephalopathy in the setting of elevated arterial ammonia levels is with oral lactulose, titrated to a dose that leads to two or three soft stools daily. Rifaximin can be administered if lactulose is not well tolerated or not sufficiently efficacious. For patients who suffer repeated episodes of acute hepatic encephalopathy, chronic maintenance administration of lactulose should be prescribed. Maintaining adequate daily nutrition is important for these patients. Daily oral probiotics, such as lactobacillus, help to maintain a healthy gut flora that aids in ammonia elimination. Low-protein diet (B) is not recommended for most patients with hepatic encephalopathy. The maintenance of appropriate and adequate nutrition is important for patients with hepatic insufficiency, and low-protein diets can be detrimental to their health. For patients with portosystemic shunts who have severe hepatic insufficiency and have suffered repeated bouts of encephalopathy secondary to high-protein intake, restriction of dietary animal protein may be necessary. These patients should be encouraged to replace the animal protein in their diet with vegetable protein. Two or three large meals daily with no snacking (C) is not appropriate advice for a patient with hepatic encephalopathy. Patients with chronic hepatic encephalopathy should be encouraged to eat several small meals throughout the day, as well as a late night snack of complex carbohydrates, because the body produces more ammonia in a fasting state. Zinc supplementation (D) has been anecdotally recommended to improve symptoms of hepatic encephalopathy, but limited clinical trials have shown no statistically significant effect on symptoms of hepatic encephalopathy with zinc supplementation. Many patients with hepatic encephalopathy do, however, have low serum levels of zinc.

A 43-year-old man reports to the orthopaedic clinic for evaluation of his right third finger. He states that this finger gets stuck when he makes a fist, and he has to manually extend it. Which of the following is likely found in this patient's medical history? A. Diabetes mellitus B. Multiple sclerosis C. Osteoarthritis D. Scleroderma

Correct Answer: (A) Diabetes mellitus Explanation: Tenosynovitis is defined as inflammation of the sheath that surrounds a tendon, which is called the synovium. It can be caused by infection or can have an idiopathic etiology. Stenosing flexor tenosynovitis is also called trigger finger and is caused by a disparity in size between the flexor tendon and the surrounding pulley system at the first annular (A1) pulley that overlies the metacarpophalangeal joint. This size disparity causes the flexor tendon to catch as it attempts to glide through the stenotic sheath with flexion and extension of the finger. Trigger finger is common and affects roughly 2% of the general population. It is more common in women in the fifth or sixth decades of life and can occur in one or multiple fingers in one or both hands. Patients with diabetes mellitus, rheumatoid arthritis, or deposition disorders (e.g., amyloidosis) have an increased risk of developing trigger finger. Patients commonly complain of catching, snapping, and an inability to extend the finger after flexion and often resort to manual extension using the other hand. While the condition is initially painless, patients may complain of pain at the base of the digit directly over the A1 pulley, which can be exacerbated with passive hyperextension. Pain can also radiate into the palm or distal finger. Physical exam may reveal a palpable nodule and clicking with flexion and extension of the affected finger. Diagnostic imaging is not useful in the diagnosis of stenosing flexor tenosynovitis. Conservative treatment of trigger finger initially includes activity modification, splinting, and short-term nonsteroidal anti-inflammatory drugs. Splints should keep the metacarpophalangeal joint in slight flexion and use should be dictated by patient preference. The splint should be used for three to six weeks. Patients refractory to conservative treatment can be treated with a local glucocorticoid injection (e.g., methylprednisolone, triamcinolone) mixed with a local anesthetic, such as lidocaine. Injections can be repeated after six weeks for a maximum of three injections in patients whose symptoms have not improved by at least 50% with this treatment. Diabetic patients are less likely to benefit from glucocorticoid injections. Extensive involvement or cases refractory to these treatments require surgical intervention via either ultrasound-guided percutaneous or open surgical release of the A1 pulley ligament. Both methods are effective with a triggering recurrence rate of 3%. Surgical complications include infection, digital nerve injury, flexor tendon bowstringing, and tendon scarring. Patients who are refractory to initial treatment or who may require surgical intervention should be referred to an orthopaedic specialist for further evaluation and treatment. Multiple sclerosis (B) does not increase the risk of trigger finger. Primarily a neurologic disorder, common symptoms of multiple sclerosis include sensory disturbance in the limbs or face, visual loss, motor weakness, diplopia, gait disturbance or balance problems, and pain. Physical exam may reveal a Lhermitte sign. A diagnosis of multiple sclerosis does not predispose an individual to developing trigger finger as seen in the patient in the vignette above. Osteoarthritis (C) may cause palpable bony nodules on the fingers but does not commonly present with catching and locking. Bouchard nodes present on the proximal interphalangeal joint while Heberden nodules appear on the distal interphalangeal joint. While the two conditions may coexist, osteoarthritis does not increase the risk of developing trigger finger. Scleroderma (D) is a disease that manifests locally with sclerotic skin lesions or systemically with involvement of the internal organs. Depending on the type of scleroderma, laboratory testing reveals anti-DNA topoisomerase I (Scl-70) antibodies with a speckled staining pattern, antinuclear antibodies, anticentromere antibodies, and anti-RNA polymerase III antibodies. The patient in the above vignette describes signs and symptoms that are consistent with a diagnosis of trigger finger, which is more common in patients with diabetes mellitus and has not been shown to be associated with scleroderma patients.

A 34-year-old woman presents to the emergency department with crampy, abdominal pain. She also reports malaise, weight loss, fatigue, and chronic, intermittent, nonbloody diarrhea. Physical exam reveals tenderness to palpation in the right lower quadrant with a palpable mass. Urine pregnancy test is negative. Ileocolonoscopic exam reveals focal ulceration and areas with normal mucosa. Which of the following is the best initial treatment? A. Enteric-coated budesonide B. Mesalamine C. Methrotrexate D. Oral vancomycin

Correct Answer: (A) Enteric-coated budesonide Explanation: Inflammatory bowel disease is comprised of Crohn disease and ulcerative colitis, which are conditions with chronic inflammation of the bowel. Crohn disease most commonly affects the terminal ileum but can cause mucosal defects in any portion of the gastrointestinal tract from mouth to anus. It has been associated with autoimmune factors, but the exact pathogenesis is not well understood. Risk factors for developing Crohn disease include age between 15 and 40, history of smoking, decreased physical activity, diet, and nonsteroidal anti-inflammatory drug use. Symptoms of Crohn disease include unintentional weight loss, intermittent low-grade fever, nonbloody diarrhea, and right lower quadrant pain. A tender, palpable mass in the lower abdomen indicates inflamed loops of bowel. Arthritis, arthropathy, episcleritis, anterior uveitis, primary sclerosing cholangitis, kidney stones, vitamin B12 deficiency, and pulmonary conditions (e.g., chronic bronchitis, interstitial lung disease) characterize the extraintestinal manifestations of disease. The most common dermatologic manifestations are pyoderma gangrenosum and erythema nodosum. Lab studies should include a CBC, CMP, albumin, erythrocyte sedimentation rate (ESR), C-reactive protein (CRP), and iron, vitamin D, and vitamin B12 levels. Specific antibodies, antineutrophil cytoplasmic antibodies (ANCA) and anti-Saccharomyces cerevisiae antibodies (ASCA), are available for diagnosis of Crohn disease, but the accuracy and predictive value of these markers remain controversial. Skip lesions are found on colonoscopy and are defined as segments of transmural inflammation with adjacent normal-appearing mucosa. High-risk patients with moderate to severe Crohn disease typically demonstrate an earlier age of onset, tobacco use, elevated C-reactive protein or fecal calprotectin levels, deep ulcers on colonoscopy, long segments of bowel involvement, perianal disease, extraintestinal manifestations, and a history of bowel resections. Both induction and maintenance of remission have two approaches in regard to management. Step-up therapy protocols begin with low potency, less toxic medications such as glucocorticoids (prednisone, budesonide) with escalation of therapy to more toxic and potent agents after initial treatment is not successful. Top-down therapy advocates use of high-potency agents (infliximab) to more quickly induce remission and reduce the risk of glucocorticoid dependence. Treatment of patients with mild, low-risk disease generally uses step-up therapy, while moderate to severe Crohn disease patients with a high risk of complications may be started on top-down therapy. Step-up therapy induces remission with enteric-coated budesonide. If remission is achieved, the glucocorticoid is tapered and discontinued, with an ileocolonoscopy performed in 6 to 12 months. Traditionally used 5-aminosalicylates (e.g., mesalamine, sulfasalazine) have mixed data on efficacy but may be used as alternative agents for low-risk mild Crohn disease. Tumor necrosis factor inhibitors (e.g., infliximab) are used in combination with azathioprine or mercaptopurine to induce remission in top-down therapy. Maintenance of remission in high-risk patients with moderate or severe Crohn disease involves indefinite use of biologic agents (infliximab) with discontinuation of thiopurine medications after 1 to 2 years. Common complications of Crohn disease include abscesses, intestinal obstruction, fistulae, and perianal disease, such as anal fissures and skin tags. Many patients ultimately require surgical management with intestinal resection to alleviate symptoms and treat complications, but surgery is not curative but rather palliative. Mesalamine (B) is a 5-aminosalicylic acid agent that has recently been shown to have little to no value in the induction or maintenance of remission for patients with Crohn disease. It should not be used as a first-line agent. Methotrexate (C) is not an appropriate first-line medication in a patient with mild Crohn disease manifestations. This medication is used in patients with moderate to severe active disease who are unresponsive to or intolerant of mercaptopurine or azathioprine. Oral vancomycin (D) is used in the treatment of Clostridioides difficile colitis.

A 10-year-old boy presents to the emergency department complaining of right eye redness, pain, photophobia, and decreased visual acuity after being struck in the face with a baseball. Physical exam reveals blood pooling in the inferior portion of the anterior chamber, obscuring the inferior one-third of the iris and inferior portion of the pupil. The visible portion of the pupil is round and reactive to light. Swinging light test is negative for afferent pupillary defect on the right. Extraocular muscle range of motion is normal bilaterally. X-ray of the skull is negative for orbital fracture. Fluorescein staining is negative for corneal abrasion. Which of the following is the most likely diagnosis? A. Hyphema B. Open-globe rupture C. Subconjunctival hemorrhage D. Traumatic iritis

Correct Answer: (A) Hyphema Explanation: Hyphema refers to gross pooling of blood in the anterior chamber of the eye. The most common cause of hyphema is traumatic eye injury, generally sports-related in younger children and more likely to be assault in adolescents and adults. The symptoms of hyphema include eye pain, redness, decreased visual acuity, photophobia, nausea, vomiting, and drowsiness. The sign of a hyphema is pooling of blood in the anterior chamber, although these injuries may also be accompanied by decreased visual acuity, anisocoria, corneal abrasion, open-globe rupture, elevated intraocular pressure, lens subluxation, and bradycardia. Open-globe rupture must be ruled out before pressure is applied to the eye during physical exam and before drops are instilled. A patient presenting with a hyphema and no history of ocular trauma should be tested for diabetes mellitus, clotting disorders, juvenile xanthogranuloma, and eye tumors. Medications that increase the risk of bleeding, such as warfarin and aspirin, can also cause nontraumatic hyphema. Treatment of hyphema requires ophthalmologic consultation, and preservation of vision is often dependent on time to treatment and hyphema grade. Blood which pools under one-third of the anterior chamber is classified as a grade I hyphema. One-third to one-half of the anterior chamber covered by pooled blood is classified as grade II hyphema. Over 50 percent but under 100 percent filling of the anterior chamber with blood is grade III, and 100 percent is grade IV. Those with grade I hyphema have a 90% rate of recovery of 20/50 vision or better with prompt management of the hyphema. Those with grade III or IV hyphema have a 50% chance of recovery of 20/50 vision. While ophthalmologic consultation is pending, the patient should be put in a quiet, dimly lit room and rest on a bed with 30 degrees of head elevation to facilitate hyphema resolution. The patient should rest and not read or exert the injured eye. A protective eye shield should be placed, antiemetics administered, and coagulopathies addressed. Analgesic eye drops and cycloplegic eye drops can be administered for pain control only after open-globe rupture has been ruled out via computed tomography. Open-globe rupture (B) refers to a full-thickness laceration of the eye surface, allowing the contents of the deeper layers to protrude. An open-globe rupture and hyphema can occur by the same mechanism, making it necessary that the provider rules out open-globe rupture in all patients who present with hyphema. Signs of open-globe rupture include eccentric pupil, streaming of fluorescein dye away from the puncture site, increased or decreased anterior chamber depth, tenting of the cornea or sclera, extravasation of vitreous, protrusion of lens or iris, and low intraocular pressure. This patient has none of these signs and a negative fluorescein stain. If globe rupture is strongly suspected, a computed tomography scan of the orbit should be obtained (but not if it would delay ophthalmologic consult). Subconjunctival hemorrhage (C) refers to bleeding beneath the conjunctiva. The blood will be limited to the conjunctiva and not be present in the anterior chamber. Causes of subconjunctival hemorrhage may mimic those of hyphema, although sneezing, prolonged coughing, and spontaneous hemorrhage of small subconjunctival vessels can also cause subconjunctival hemorrhage. Traumatic iritis (D) refers to inflammation of the iris due to trauma. Patients with traumatic hyphema may also have traumatic iritis, but traumatic iritis alone does not include anterior chamber bleeding. Patients with traumatic iritis will have white blood cells and flare seen on slit-lamp exam and may even having pooling of white blood cells in the anterior chamber, known as hypopyon.

A 22-year-old man presents to the ED after falling while skateboarding. The patient was initially alert and talking to you, but is becoming progressively more somnolent. A non-contrast computed tomography scan of his head is shown above. Which of the following pairs represents the correct diagnosis and most likely injured vessel? A. Epidural hematoma, bridging veins B. Epidural hematoma, middle meningeal artery C. Subdural hematoma, bridging veins D. Subdural hematoma, middle meningeal artery

Correct Answer: (B) Epidural hematoma, middle meningeal artery Explanation: This patient's imaging and physical examination are consistent with a traumatic acute epidural hematoma. An epidural hematoma is a collection of blood between the skull and the dura. Epidural hematomas are usually associated with skull fractures in the temporal bone region resulting in laceration of the middle meningeal artery. Arterial bleeding is the etiology of ⅔ of epidural hematomas and is typically rapid. Epidural hematomas are uncommon in the elderly and in children < 2 years of age due to the close attachment of the skull to the dura. Epidural hematomas account for only 1% of all head-injured patients presenting with coma and are present in 0.5% of all head-injured patients. Signs and symptoms include severe headache, drowsiness, nausea, and vomiting. The classic finding is the lucid interval just prior to rapid deterioration; however, this is present in < 30% of epidural bleeds. Diagnosis is via non-contrast CT of the head. This characteristically shows a hyperdense lenticular-shaped hematoma in the temporal region. These hematomas are sharply defined and do not cross suture lines. Management of epidural hematomas is primarily surgical. Neurosurgery should be consulted immediately for surgical evacuation of the hematoma. Prognosis is good if the epidural hematoma is promptly treated and the patient does not present with coma. Epidural hematoma, bridging veins (A) is an incorrect pairing as the imaging shows an epidural hematoma, however, this is typically caused by injury to the middle meningeal artery and not tearing of the bridging veins. Subdural hematoma, bridging veins (C) is a correct pairing; however, it is an incorrect answer as the imaging shows an epidural hematoma. Subdural hematoma, middle meningeal artery (D) is an incorrect pairing as the imaging shows an epidural hematoma and subdural hematomas are caused by tearing of the bridging veins.

A 55-year-old man with a history of myocardial infarction presents to the emergency department with worsening dyspnea and fatigue, especially with activity. Upon physical examination, the S1 heart sound is diminished and there is a high-pitched, blowing, holosystolic murmur auscultated over the apex radiating to the left axilla. The murmur does not change with inspiration or with changes in position. A chest radiograph reveals left ventricular enlargement. Which of the following is the most likely diagnosis? A. Aortic stenosis B. Mitral regurgitation C. Tricuspid regurgitation D. Ventricular septal defect

Correct Answer: (B) Mitral regurgitation Explanation: Mitral regurgitation occurs when there is an abnormal reversal of blood flow from the left ventricle to the left atrium caused by a disruption of any part of the mitral valve. It is the most common valvular disease. Mitral valve prolapse, rheumatic heart disease, infective endocarditis, annular calcification, cardiomyopathy, and ischemic heart disease are common etiologies of this condition. Congenital mitral regurgitation is rare. Since it has multiple causes, the pathophysiology, clinical manifestations, and management of this condition depend on the etiology, severity, and chronicity of the disease. When mitral regurgitation is acutely associated with coronary artery disease and myocardial infarction, it is considered a medical emergency. Patients will present with hemodynamic compromise with symptoms such as dyspnea, fatigue, orthopnea, and pulmonary edema, all as a result of impaired left ventricular dysfunction. Patients with chronic mitral regurgitation may be asymptomatic for several years until systolic left ventricular dysfunction develops, resulting in reduced cardiac output due to progressive enlargement of the left atrium and ventricle. Patients will then experience decreased exercise tolerance due to decreased cardiac output, chest palpitations (if atrial fibrillation develops), progressive left ventricular enlargement leading to congestive heart failure, and pulmonary congestion and edema. On physical examination, a brisk carotid upstroke and hyperdynamic cardiac impulse may be palpated. Upon auscultation, there may be a diminished S1, a wide splitting S2 (as a result of early closure of the aortic valve), an audible S3 (due to left ventricular dysfunction or increased blood flow across the mitral valve), and an accentuated P2 (if pulmonary hypertension is present). There may also be a characteristic high-pitched, blowing, and holosystolic murmur present that is best heard over the apex radiating to the left axilla. Exercise testing is useful for assessing functional status and eliciting symptoms. A chest radiograph may reveal left ventricular enlargement due to volume overload (especially seen in chronic mitral regurgitation), left atrial enlargement, and pulmonary congestion if congestive heart failure has developed. A transthoracic echocardiogram (TTE) should be ordered to evaluate for baseline left ventricular size and function, right ventricular and left atrial size, pulmonary arterial pressure, and severity of the condition. It is also helpful in determining the etiology of mitral regurgitation. Patients with moderate-to-severe mitral valve regurgitation should have a TTE every 6-12 months for surveillance. Other tests include electrocardiography (to assess for ischemia, infarction, left ventricular hypertrophy, and atrial fibrillation), brain natriuretic peptide, and cardiac catheterization. Optimal treatment of mitral regurgitation depends on etiology, chronicity, severity, and patient comorbidities. Any patient with hemodynamic compromise should be evaluated for acute myocardial infarction for early intervention and close monitoring. Diuretics (e.g., furosemide) should be given to those with pulmonary congestion, and afterload-reducing agents (e.g., nitrates, angiotensin-converting enzyme inhibitors) are helpful in maintaining cardiac output. Surgical mitral valve repair is the criterion standard for intervention of moderate-to-severe mitral regurgitation, or those with left ventricular dysfunction. Aortic stenosis (A) can be confused with mitral regurgitation because it produces a prominent murmur at the apex but it is characterized classically by a crescendo-decrescendo systolic murmur, not a holosystolic murmur. It is also has a harsh quality, is best heard over the right upper sternal border, and radiates to the carotids. Tricuspid regurgitation (C) also causes a holosystolic murmur, however, it is located at the left lower sternal border, not the apex. It also does not radiate to the axilla, and it increases in intensity with inspiration whereas mitral regurgitation does not. A ventricular septal defect (D) produces a harsh holosystolic murmur at the lower left sternal border and is typically associated with a palpable thrill. It radiates to the right of the sternum, not the axilla.

A 24-year-old nurse sustains a needlestick injury while working in the hospital. He has not completed his hepatitis B vaccination series. The source patient agrees to a hepatitis B serology panel, which reveals positive hepatitis B surface antigen, negative hepatitis B surface antibody, positive hepatitis B core antibody, and positive immunoglobulin M hepatitis B core antibody. Which of the following is the appropriate treatment for the exposed nurse? A. No prophylaxis needed B. One dose each of hepatitis B immunoglobulin and hepatitis B vaccine C. One dose of hepatitis B immunoglobulin with a second dose in one month D. One dose of hepatitis B vaccine

Correct Answer: (B) One dose each of hepatitis B immunoglobulin and hepatitis B vaccine Explanation: Hepatitis B is a viral infection that causes acute and chronic liver disease and can result in cirrhosis and end-stage liver failure. This infection is transmitted from infected individuals to those who are not immune via mother-to-child transmission and exposure to bodily fluids through transfusion, sexual contact, percutaneous inoculation, and transplantation of organs, among other means. Health care providers are at particular risk for contracting hepatitis B due to their regular occupational exposure to blood and bodily fluids. A needlestick injury carries the highest risk of developing hepatitis B virus infection. All providers, therefore, are strongly encouraged to be immunized against this pathogen. Immunization in adults consists of three doses of the conventional vaccine with the second and third doses administered one and six months after the first dose. Health care providers should be tested for immunity one to two months after receiving the last dose. An appropriate immune response is indicated by a hepatitis B surface antibody (anti-HBs) ≥ 10 mIU/mL. An additional dose can be administered to providers who do not meet this level of response with repeat testing. Some individuals require up to three additional doses of the vaccination to demonstrate adequate antibody response. If anti-HBs values do not meet required minimum levels, the health care provider is considered a nonresponder and should receive hepatitis B immune globulin (HBIG) if post-exposure prophylaxis is required. If a health care provider is exposed to hepatitis B virus, the infection status of the source patient should be obtained. If unknown, the source patient should be presumed infectious. A hepatitis B serologic panel indicates the presence of hepatitis B surface antigen (HBsAg), hepatitis B core antibody (anti-HBc), hepatitis B surface antibody (anti-HBs), and immunoglobulin M hepatitis B core antibody (IgM anti-HBc). A patient with a positive HBsAg is infectious, with positive IgM anti-HBc indicating an acute versus a chronic infection. Positive anti-HBc with negative HBsAg indicates immunity conferred by prior infection with hepatitis B. Vaccination results in positive anti-HBs but negative anti-HBc, as antibody response to the core antigen can only be elicited with live virus infection. A patient with no detectable hepatitis B surface antibodies or a titer level of less than 10 mIU/mL is susceptible to infection. Health care providers with an adequate vaccine response do not require post-exposure prophylaxis. Those with low titer levels and exposure to an infectious patient require one dose of hepatitis B immunoglobulin and a dose of the hepatitis B vaccine. The vaccination series should then be completed according to guidelines. If the source patient is not infectious, providers with low titer levels should receive a single dose of the vaccine with repeat antibody testing in one to two months. Providers who have not completed the vaccine series should not be tested for antibody presence and should instead be treated with a dose of hepatitis B immunoglobulin and the first dose of the hepatitis B vaccine series if the patient is infectious. Exposure to noninfectious patients warrants completion of the vaccination series in health care providers. Follow-up testing should include hepatitis B core antibody and hepatitis B surface antigen testing six months after exposure to determine transmission. No prophylaxis is needed (A) is an inaccurate statement due to the infectious state of the source patient, indicated by a positive hepatitis B surface antigen result. Health care providers who are nonresponders are prophylactically treated after needlestick exposure to a patient with HBsAg positive or unknown results with one dose of hepatitis B immunoglobulin with a second dose in one month (C). One dose of the hepatitis B vaccine (D) would not be adequate to prophylactically treat this health care provider as he has not completed the hepatitis B vaccination series and the source patient is infectious. Health care providers who have an unknown immunization status and demonstrate a hepatitis B surface antibody titer < 10 mIU/mL should receive this prophylactic treatment if the source patient is HBsAg negative.

A 25-year-old man presents to the clinic with muscle weakness, headache, polyuria, and polydipsia. His medical history includes hypertension, for which he takes hydrochlorothiazide, amlodipine, and enalapril. Vitals show HR 82 bpm, RR 16/min, T 98.3°F, BP 152/103 mm Hg, and SpO2 99% on room air. Review of past visits reveals similar blood pressure values. A basic metabolic panel reveals a serum potassium of 2.9 mmol/L and a serum bicarbonate of 25 mmol/L. Which of the following tests confirms this patient's underlying disorder? A. Hypertonic saline infusion test B. Oral sodium loading test C. Serum aldosterone D. Serum bicarbonate

Correct Answer: (B) Oral sodium loading test Explanation: Hypertension is an elevation of systemic blood pressure evidenced by a systolic value above 140 mm Hg or a diastolic value above 90 mm Hg. The most common form of hypertension is essential or primary hypertension. Secondary hypertension occurs as a result of another underlying condition, which is most commonly renovascular disease. Primary aldosteronism, also called hyperaldosteronism, is the most common cause of refractory hypertension in youth and middle-aged adults and occurs due to an inappropriately high aldosterone secretion that does not suppress with sodium loading, which results in sodium retention and suppression of plasma renin. Hyperaldosteronism is caused by a unilateral adrenal adenoma or hyperplasia or by bilateral adrenal hyperplasia. Hypertension is usually moderate in patients with this condition but may be severe. Diastolic hypertension may be the only clinical manifestation, and edema is rarely associated. Hypokalemia can result in muscle weakness, paresthesias with frank tetany, headache, polyuria, and polydipsia. Routine laboratory findings commonly include hypokalemia and elevated serum bicarbonate. Specific testing is indicated in patients with sustained hypertension above 150/100 mm Hg on three separate days, hypertension resistant to three conventional antihypertensive medications (including a diuretic), controlled blood pressure requiring four or more drugs, hypokalemia, a personal or family history of early-onset hypertension or cerebrovascular accident at age 40 or younger, a first-degree relative with the condition, presence of an adrenal mass, or low plasma renin activity. An oral sodium loading test involves two weeks of a high-sodium diet and removal of certain medications (i.e., diuretics, angiotensin-converting enzyme inhibitors, angiotensin receptor blockers, beta-blockers, clonidine, nonsteroidal anti-inflammatory drugs, oral estrogens, and oral contraceptives). The plasma renin activity or a direct renin assay are then drawn using a specific protocol. Plasma renin activity levels of < 0.65 ng/mL/h or direct renin assay < 0.36 ng/mL is diagnostic of primary aldosteronism while a serum aldosterone to plasma renin activity ratio < 24 excludes the diagnosis. A ratio greater than 67 can also be used to support the suspected diagnosis as it is highly suggestive. Further confirmation of the diagnosis is acquired via a 24-hour urine collection in an acidified container, which is tested for aldosterone, cortisol, and creatinine. Urinary aldosterone levels greater than 20 mcg/24 h confirms primary aldosteronism. All patients with confirmed diagnosis should undergo genetic testing and a thin-section CT scan of the adrenals. Primary aldosteronism that is caused by a unilateral aldosteronoma can be cured with surgical resection. Those with disease secondary to bilateral hyperplasia are instead treated medically with a potassium-sparing diuretic, particularly eplerenone, spironolactone, or amiloride. Other antihypertensive medications such as amlodipine, angiotensin-converting enzyme inhibitors (enalapril, captopril), or angiotensin receptor blockers (losartan) may be required for adequate blood pressure control. Hypertension is reversible in two-thirds of patients. Cardiovascular complications such as stroke occur more commonly in patients with primary hyperaldosteronism than essential hypertension. An isotonic saline infusion test, not a hypertonic saline infusion test (A), may be used as an alternative confirmation test if the patient is unable to undergo oral sodium loading. This test involves the intravenous administration of 2 L of isotonic saline over 4 hours. Primary aldosteronism is confirmed with plasma aldosterone concentrations above 10 ng/dL. Serum aldosterone (C) may be used as a precursor test in patients with suspected hyperaldosteronism but does not confirm the diagnosis. Serum bicarbonate (D) is often elevated in patients with primary aldosteronism but is not a confirmation of the disease, as this value only indicates metabolic alkalosis.

A 23-year-old woman presents to the emergency department with worsening lightheadedness, dizziness, and one fainting episode that occurred yesterday. An electrocardiogram is ordered, which reveals a gradually progressive PR interval prolongation followed by a blocked P wave and a dropped QRS complex. Which of the following is the most likely diagnosis? A. First-degree AV block B. Second-degree Mobitz Type I AV block C. Second-degree Mobitz Type II AV block D. Third-degree AV block

Correct Answer: (B) Second-degree Mobitz Type I AV block Explanation: Second-degree atrioventricular (AV) block, also known as second-degree heart block, is a disease affecting the conduction system of the heart in which the atrial impulse through the AV node is delayed or blocked. The blockage may or may not include the bundle of His. The location of the block, whether or not it is in the AV node or infranodal system (i.e., the His-Purkinje conduction system), is significant for prognosis. AV nodal blocks carry a more favorable prognosis since infranodal blocks have risk of progressing to complete heart block. There are two types of second-degree AV blocks: Mobitz Type I (also known as Wenckebach block), which is the more common form, and Mobitz Type II, which is a block in or below the bundle of His. The latter type may progress to a complete heart block, which has an associated risk of mortality. A main cause for AV block are cardioactive drugs, such as digoxin, beta-blockers, calcium channel blockers, and certain antidysrhythmic drugs, which may act to exert negative effects on the AV system. Other drugs such as lithium, benzathine penicillin, and alpha agonists have been implicated in second-degree AV block. This condition may also be associated with underlying diseases such as inflammatory diseases (e.g., endocarditis, myocarditis, Lyme disease), infiltrative disease (e.g., amyloidosis), malignancies, metabolic and endocrine disorders (e.g., hyperkalemia), and collagen vascular diseases (e.g., ankylosing spondylitis, scleroderma). It is also associated with history of myocardial ischemia and infarction, cardiac tumors, trauma, ethanol poisoning, and iatrogenic heart procedures. Some patients have a genetic predisposition and may have underlying structural heart disease. The rate of occurrence is higher in trained athletes. An electrocardiogram is used to diagnose the presence and type of second-degree AV block. Mobitz Type I AV block is characterized by a gradually progressive PR interval prolongation followed by a blocked P wave impulse (and therefore a dropped QRS complex), with the greatest PR increment typically occurring between the first and second beats of the cycle and gradually decreasing in subsequent beats. This occurs because the atrial impulse fails to conduct, leading to a QRS complex not being generated, and a ventricular contraction not being produced. There is a shortening of the PR interval that occurs after the blocked sinus impulse, which is characteristic of Mobitz I block. There may be junctional escape beats that occur along with the nonconducted P waves. There is a pause that occurs after the blocked P wave that is less than the sum of the two beats before the block. During very long sequences, PR-interval prolongation may not be apparent until the last beat of the cycle. R-R intervals shorten as PR intervals become longer. A Mobitz II AV block is characterized by an unexpected nonconducted atrial pulse (loss of P wave) without prior measurable lengthening of the conduction time. Typical ECG findings include consecutively conducted beats with the same PR interval followed by a blocked sinus P wave. There is no change in the PR intervals during consecutive beats before the dropped P wave. Patients with second-degree AV block may be asymptomatic (more commonly in patients with type I disease) or have a variety of symptoms such as lightheadedness, dizziness, hypotension, diaphoresis, bradycardia, and syncope. Some patients may experience palpitations, or if there is an associated myocardial ischemia or infarction, may present with chest pain. Laboratory studies may be ordered to determine underlying causes such as serum electrolytes, calcium, magnesium, digoxin levels (for those who are taking it), cardiac biomarker testing (for those with suspected ischemia), thyroid function, and certain viral titers if myocarditis is suspected. Patients who have Mobitz Type I second-degree AV block who are asymptomatic without underlying etiology usually do not require treatment. Patients who are symptomatic should be treated with atropine and transcutaneous pacing followed by transvenous temporary pacing until etiology of disease is determined. If patients have concomitant acute myocardial ischemia or infarction, they should be admitted and treated immediately. Patients with Mobitz Type II second-degree AV block should be admitted and closely monitored. Transcutaneous pacing should be started in all patients with this condition, including those who are asymptomatic due to risk of progression to complete heart block. Permanent pacing for those with second-degree heart block is indicated in those who have bradycardia, heart failure, and asystole for three seconds or longer while awake, those with Mobitz II with wide QRS complexes, and those who have Mobitz I with a block within the bundle of His. It is also indicated in those with concomitant neuromuscular disease such as myotonic muscular dystrophy whether or not they are symptomatic. First-degree atrioventricular block (A) is characterized by a persistent PR interval that is greater than 0.2 seconds. The PR interval does not change in length. Second-degree Mobitz Type II atrioventricular block (C) is characterized by consecutively conducted beats with the same PR interval followed by a blocked sinus P wave. There is no change in the PR intervals during consecutive beats before the dropped P wave. In third-degree atrioventricular block (D), or a complete heart block, the P waves are not related to the QRS complexes due to the atria being electrically disconnected from the ventricles.

A 22-year-old woman with normal body mass index presents to the emergency department complaining of weakness and fatigue. She is found to be hypokalemic and alkalotic, with dental enamel erosions. Upon questioning, the patient reports binge eating and self-induced vomiting at least twice daily for the past 6 months. Which of the following represents the best choice of pharmacotherapy for this patient after electrolyte imbalances have been corrected? A. Amitriptyline B. Buproprion C. Fluoxetine D. Topiramate

Correct Answer: (C) Fluoxetine Explanation: Bulimia nervosa is an eating disorder characterized by excessive calorie intake, followed by self-induced purging of calories (via vomiting, diuretics, laxatives, fasting, or excessive exercise). Patients with bulimia nervosa have an altered self-image that is overly concerned with weight and body shape. For diagnosis, the binging and purging episodes must occur at least once weekly for 3 months or longer. During an episode of binge eating, the patient may describe feeling out of control. The exact etiology of bulimia nervosa is not known, although disordered serotonin pathways play a role. Women are more likely to suffer bulimia nervosa than men, and the average age of onset is 20 years. Patients with bulimia nervosa often have comorbid psychological disorders, such as major depressive disorder, anxiety disorders, post-traumatic stress disorder, and body dysmorphic disorder. Cutting and other nonsuicidal self-injurious behaviors are often seen in patients with bulimia nervosa. Complications of bulimia nervosa include electrolyte imbalances, dehydration, dental erosions, Mallory-Weiss esophageal mucosal lacerations, ipecac-induced myopathy, endocrine dysfunction, and cardiac dysrhythmias (rare). Diagnosis of bulimia nervosa is clinical, as described above, based on the Diagnostic and Statistical Manual of Mental Disorders, volume 5, guidelines. The binge episode must include excessive calorie consumption in a discrete period of time, accompanied by feelings of loss of control. The eating disturbance must not occur exclusively during an episode of anorexia nervosa. Treatment of bulimia nervosa is with a three-pronged approach of nutritional rehabilitation, psychotherapy, and pharmacotherapy. Either of the three may be used alone, but a combination of all three has proven most effective. In nutritional rehabilitation, the patient is encouraged to consume an appropriate amount of calories throughout the day, including three meals and two snacks. Psychotherapy focuses on body image, feelings of loss of control, and concomitant depression, anxiety, or other psychological disturbances. Pharmacotherapy with antidepressants has proven effective in the treatment of bulimia nervosa, possibly due to the disruption of serotonin pathways found in the illness. Selective serotonin reuptake inhibitors, such as fluoxetine, are first-line therapy due to their minimal effects on weight and favorable side effect profile. Patients should be observed closely during the first few weeks of treatment, as this class of medicine is associated with an increased risk of suicidality in young patients. Amitriptyline (A) is a tricyclic antidepressant that has good efficacy in bulimia nervosa but an undesirable side effect profile, including somnolence and weight gain. Patients who already are overly focused on weight and body shape are not likely to adhere to a regimen that causes noticeable weight gain. Although the selective serotonin reuptake inhibitors can cause weight gain, the gain tends to be slow and minimal as compared to tricyclic antidepressants or monoamine oxidase inhibitors. Bupropion (B) is contraindicated in bulimia nervosa due to an increased incidence of seizures when administered to this patient population. Topiramate (D) is an anticonvulsant that is also used as a mood stabilizer. Topiramate has shown modest therapeutic effects in patients with bulimia nervosa. A side effect of topiramate is weight loss, so this drug should be used with caution in bulimia nervosa, as the weight loss may be a trigger for increased body dysmorphia or disordered thoughts about body shape. Generally, topiramate is used as a third-line agent in the treatment of bulimia nervosa after fluoxetine and sertraline.

A 45-year-old woman presents to the emergency department with recurrent, spontaneous bruising and nosebleeds. Physical examination reveals purpura and petechiae on the lower extremity bilaterally but is otherwise normal. Laboratory studies reveal a platelet count of 20,000/mcL and normal prothrombin and activated partial thromboplastin times. Which of the following is the most likely diagnosis? A. Hemolytic uremic syndrome B. Heparin-induced thrombocytopenia C. Idiopathic thrombocytopenic purpura D. Thrombotic thrombocytopenic purpura

Correct Answer: (C) Idiopathic thrombocytopenic purpura Explanation: Idiopathic thrombocytopenic purpura (also called primary immune thrombocytopenia or immune thrombocytopenic purpura) is an autoimmune bleeding disorder in which pathogenic antibodies bind platelet antigens, such as glycoproteins IIb/IIIa and Ib/IX, thereby facilitating their clearance from the circulation. The principal mechanism of idiopathic thrombocytopenic purpura involves specific immunoglobulin G produced by the B cell. Idiopathic thrombocytopenic purpura may be acute or chronic and primary or secondary. The acute form that is self-limiting is primarily seen in children after a viral illness or immunization. The prevalence of the chronic form is significantly higher in adults, especially women of childbearing age. While idiopathic thrombocytopenic purpura may be primary in most adults, it may be associated with connective tissue diseases (such as systemic lupus erythematosus), lymphoma, medications, and infections (such as the hepatitis C virus and HIV). Clinical manifestations depend on the platelet count and may include spontaneous bruising and nosebleeds, gingival bleeding, purpura, petechiae, menorrhagia, and intracranial bleeding in severe cases. Laboratory studies typically show isolated thrombocytopenia, normal prothrombin time, and normal activated partial thromboplastin time but may also show anemia in the setting of bleeding. Bone marrow may show elevated megakaryocytes but is rarely needed for diagnosis unless in patients with unexplained cytopenias in two or more lineages, patients older than 40 years, or in those who have failed the initial therapy for idiopathic thrombocytopenic purpura. Treatment is reserved for patients with a platelet count of 20,000 to 30,000/mcL or those with significant bleeding. The initial treatment of new-onset idiopathic thrombocytopenic purpura in adults is a short course of steroids with or without intravenous immunoglobulin or anti-D immunoglobulin. In children, watchful waiting or pharmacologic intervention using methylprednisolone or intravenous immunoglobulin may be used depending on the severity of the bleeding symptoms. Platelets may be transfused in the setting of active bleeding. Splenectomy is indicated in refractory cases. For adult patients with chronic idiopathic thrombocytopenic purpura who have failed corticosteroid therapy, immunoglobulin therapy, and splenectomy, romiplostim and eltrombopag are indicated. Hemolytic uremic syndrome (A) is a thrombotic microangiopathy associated with shiga toxin-mediated endothelial cell damage that is often contracted after ingestion of beef contaminated with Escherichia coli (types O157:H7 or O145). Individuals with hemolytic uremic syndrome are often quite ill. They typically present with hemolytic anemia, renal disease, and thrombocytopenia. Heparin-induced thrombocytopenia (B) is a form of drug-induced thrombocytopenia associated with potentially life-threatening arterial and venous thrombosis. Like individuals with idiopathic thrombocytopenic purpura, patients with heparin-induced thrombocytopenia appear quite well. However, these patients have a temporal exposure to heparin that individuals with idiopathic thrombocytopenic purpura do not have. Thrombotic thrombocytopenic purpura (D), like hemolytic uremic syndrome, is a thrombotic microangiopathy. Unlike hemolytic uremic syndrome, it is associated with autoantibodies against ADAMTS-13, a von Willebrand factor-cleaving protease. Individuals with thrombotic thrombocytopenic purpura typically present with fever, hemolytic anemia, thrombocytopenia, renal disease, and neurological deficits.

A 65-year-old man with recent history of myocardial infarction presents to the emergency department with palpitations, lightheadedness, and hypotension. He seems to have an altered level of consciousness. An electrocardiogram rhythm strip shows polymorphic wide QRS complexes with a rhythm of 152 beats per minute. Which of the following is the most appropriate next step for management of this condition? A. Establish IV access and administer an adenosine bolus B. Establish IV access and administer procainamide C. Immediate defibrillation D. Immediate synchronized direct current cardioversion

Correct Answer: (C) Immediate defibrillation Explanation: Ventricular tachycardia (VT) is the leading cause of sudden cardiac deaths in the United States. It is characterized by a rhythm faster than 120 beats per minute with three or more irregular beats in a row, arising from the conduction system distal to the bundle of His. The rhythm arises from the ventricular myocardium, the distal conduction system, or both. During VT, cardiac output is reduced due to decreased ventricular filling from the rapid heart rate, and lack of coordinated atrial contraction. Diminished cardiac output results in reduced myocardial perfusion, ventricular fibrillation, and sudden death. It is most commonly associated with ischemic heart disease where the myocardial scar tissue allows for electrical reentry. It is also associated with dilated cardiomyopathy, hypertrophic cardiomyopathy, Chagas disease, and surgical procedures on the ventricle. It can also be triggered by electrolyte deficiencies (e.g., hypokalemia, hypomagnesemia, hypocalcemia), systemic diseases (e.g., systemic lupus erythematosus), sympathomimetic agents (e.g., cocaine), digitalis toxicity, and drugs that cause QT interval prolongation. Patients will present with palpitations, lightheadedness, syncope, chest pain, and anxiety. Physical exam findings will reveal hypotension, tachypnea, signs of diminished cardiac output and perfusion (e.g., pallor, diaphoresis, loss of consciousness), and an elevated jugular venous pulse. Electrocardiography (ECG) is the criterion standard for diagnosis of VT, with a 12-lead ECG being obtained before rhythm conversion is initiated. However, if the patient is unstable, diagnosis can be made by physical exam and ECG rhythm strip alone. The appearance of VT on ECG is characterized by QRS complexes that are identical from beat to beat (monomorphic VT) or QRS complexes that change from beat to beat (polymorphic VT). The QRS complexes will be greater than 120 milliseconds (usually between 80 -100 milliseconds) with a pulse of greater than 100 beats per minute (usually typically around 150-200 beats per minute). All VT patients should have serum electrolytes assessed including ionized calcium, magnesium, phosphate, and potassium levels. Other laboratory studies that are helpful include testing for levels of certain high-risk drugs (e.g., digoxin), toxicology screens (for recreational drugs), and serum cardiac markers (to evaluate for myocardial ischemia or infarction). Management of VT depends on the stability of the patient, with unstable patients described as those with hypotension, decreased level of consciousness, shock, or chest pain. Stable patients may be treated with intravenous or intraosseous adenosine, and if adenosine is ineffective, procainamide or amiodarone should be used. Unstable patients with monomorphic VT should be immediately treated with synchronized direct current cardioversion, whereas unstable polymorphic VT is treated with immediate defibrillation. Cardioversion does not prevent the recurrence of VT. Long-term treatment includes antidysrhythmic drugs (e.g., amiodarone), implantable cardioverter-defibrillators, and radiofrequency ablation, depending on the underlying cause. Establish intravenous access and administer an adenosine bolus (A) is incorrect because the patient is hemodynamically unstable. Establish intravenous access and administer procainamide (B) is incorrect because the patient is hemodynamically unstable. Procainamide is also second line to adenosine when treating stable patients with VT. Immediate synchronized direct current cardioversion (D) is only indicated when the patient has monomorphic VT, not polymorphic VT.

A 27-year-old female presents to the emergency department with complaints of progressively worsening vision in her right eye over the past week. She reports right eye pain that is worse with eye movement. Her color vision is affected. She has been having flashes of light triggered by eye movement. Her physical examination demonstrates poor central visual acuity: OS 20/15, OD 20/60. Fundoscopic examination is unremarkable. Magnetic resonance imaging of the brain and orbits with gadolinium is obtained, which reveals inflammation of the optic nerve and numerous periventricular ovoid white matter lesions > 3 mm in size. Based on the imaging of the brain, what is the most likely diagnosis? A. Acute angle-closure glaucoma B. Migraine headache C. Multiple sclerosis D. Neuroretinitis

Correct Answer: (C) Multiple sclerosis Explanation: Multiple sclerosis is the most common immune-mediated inflammatory demyelinating disorder of the central nervous system. The disease can be relapsing-remitting, primary progressive, or secondary progressive. Multiple sclerosis occurs in approximately half of patients who develop optic neuritis. Optic neuritis is an inflammatory demyelinating condition involving the optic nerve. Patients present with acutely worsening vision in one eye. It is often a presenting symptom of multiple sclerosis. It is more common in women and often presents between the ages of 20-40 years old. Symptoms can include fatigue, weakness, paresthesias, visual disturbance, incoordination, imbalance, bladder dysfunction, sexual dysfunction, and gait dysfunction. Certain presenting syndromes, such as optic neuritis, transverse myelitis, and internuclear ophthalmoplegia are concerning for multiple sclerosis. These patients should have complete diagnostic workup and are at increased risk of developing the disease in the future. Physical examination findings in patients with multiple sclerosis often demonstrate brisk deep tendon reflexes, dysdiadochokinesia, positive Babinski sign, clonus, dysmetria, and nystagmus. Diagnostic workup should include magnetic resonance imaging of the brain with gadolinium. Imaging will demonstrate ovoid-shaped plaques surrounding the corpus callosum (Dawson fingers) and periventricular areas. Cervical spine and thoracic spine imaging should also be considered to evaluate for any demyelinating lesions within the cord. Lumbar puncture may be used to assess cerebrospinal fluid and for evidence of oligoclonal bands, elevated protein, and increased immunoglobulin G. Disease-modifying therapies are used to reduce the risk of overall disability and increase time in between relapses. These therapies include oral, injection, and intravenous options. In the event of an acute multiple sclerosis exacerbation, patients may undergo intravenous corticosteroids or plasma exchange if steroids are unsuccessful or not tolerated. Symptomatic treatment, including therapy and pain management, is important to address symptoms of spasticity, urinary symptoms, sexual dysfunction, and fatigue. Acute-angle glaucoma (A) is incorrect. This would not cause lesions on brain imaging, and patients often have conjunctival redness on exam. Migraine headache (B) is incorrect. While migraines can cause visual disturbance and discomfort behind the eye, white matter changes associated with migraine headaches do not commonly have this distribution on MRI imaging. Neuroretinitis (D) is incorrect. Neuroretinitis would have an abnormal fundoscopic exam.

A 69-year-old woman presents to the emergency department with a headache that is worse in the early morning for two days. She has no significant medical history. She reports a pulsatile machine-like sound in her ears. Bilateral funduscopic exam is performed, and retinal findings are shown above. Visual field and visual acuity testing are normal. Which of the following is the most likely diagnosis? A. Anterior ischemic optic neuropathy B. Central retinal vein occlusion C. Papilledema D. Papillitis

Correct Answer: (C) Papilledema Explanation: Papilledema is a term used to describe optic disc swelling due to increased intracranial pressure. The increase in pressure causes axonomic flow obstruction within the nerve, resulting in axon swelling and leakage of water, proteins, and other cellular contents into the extracellular space of the optic disc giving rise to optic disc swelling. Conditions that cause increased intracranial pressure, which may lead to papilledema, include intracranial mass lesions, cerebral edema, increased cerebrospinal fluid, pseudotumor cerebri, and obstructive hydrocephalus. Patients with papilledema usually present with bilateral optic disc swelling, a headache that is worse in the early morning, and binocular horizontal diplopia. Patients may also report a pulsatile machine-like sound in the ears. Visual symptoms are a rare finding in acute papilledema. However, untreated chronic papilledema may lead to progressive vision loss and even blindness. Fundoscopic findings in papilledema may evolve over time and are classified as early (spontaneous venous pulsation), fully developed (optic disc elevation, cup obliteration, and disc margin blurring), and chronic or late (hard exudates, flattened nerve, and disc pallor). Neuroimaging using computed tomography or magnetic resonance imaging is promptly indicated if papilledema is present on fundoscopy. If neuroimaging is normal, lumbar puncture is warranted. Other tests that may be done in the setting of papilledema include visual field testing, fluorescein angiography, and optical coherence tomography. Treatment of papilledema involves treating the underlying etiology. Anterior ischemic optic neuropathy (A) is typically unilateral and is predominantly seen in older individuals. It is often associated with hypertension and diabetes. There is always an abrupt onset of vision loss and the optic disc swelling is not as congested as seen with papilledema. Additionally, when hemorrhage is present, it is usually splinter shaped on the edge of the disk. Individuals with central retinal vein occlusion (B) usually present with vision loss depending on the severity of the hemorrhage. These individuals also have a history of diabetes, hypertension, or other atherosclerotic risk factors. Fundoscopy typically shows tortuous and dilated retinal vein, the classic "blood and thunder" appearance. Papillitis (D) is optic neuropathy in the anterior portion of the optic nerve. Patients with papillitis usually present with unilateral vision loss. Physical exam typically reveals abnormal pupillary light reflex and ocular pain with eye movement. Fundoscopy may show optic nerve congestion accompanied by yellow-white linear deposits pointing like spokes of a wheel around the fovea of the retina.

A 25-year-old man in South Florida presents during the summer with hypopigmented lesions on his upper back and proximal arms. The lesions do not itch or hurt, but he is bothered by the appearance of the lesions. What is the most likely diagnosis? A. Pityriasis rosea B. Seborrheic dermatitis C. Tinea versicolor D. Vitiligo

Correct Answer: (C) Tinea versicolor Explanation: Tinea versicolor is a common fungal skin infection that is caused by the Malassezia yeast. Risk factors include living in a tropical climate, immunosuppression, hyperhidrosis, and the use of topical oils. Tinea versicolor is not contagious. The clinical presentation of tinea versicolor varies. The lesions may be hypopigmented, hyperpigmented, or erythematous. The lesions are typically macules or thin plaques but can coalesce into larger patches. Most of the time, there are no associated symptoms, but the lesions may mildly itch. Tinea versicolor may primarily be a cosmetic concern for patients. The lesions are located on the trunk and upper extremities most often and less often on the face and neck. The diagnosis is suspected clinically but should be confirmed with a potassium hydroxide preparation. The classic potassium hydroxide preparation finding is hyphae and yeast cells, which may be described as having a spaghetti and meatballs appearance under a microscope. In one-third of cases, there is fluorescence seen with a Wood lamp. Topical antifungals are the first-line treatment. Possible agents include topical azoles (ketoconazole, clotrimazole, miconazole), topical ciclopirox, and topical selenium sulfide. Oral antifungals, such as itraconazole and fluconazole, can be used for cases with widespread involvement or in patients who fail topical therapy. It is important to educate patients that the abnormal skin pigmentation may not resolve until up to several months after successful treatment. Pityriasis rosea (A) presents with inflammatory macules and small patches in a Christmas-tree-like distribution on the trunk. There is often a larger herald patch that precedes the eruption by a couple days or weeks. Lesions are erythematous, have a scale, and may itch. The hypopigmentation and lack of history of a herald patch help to distinguish these conditions in the vignette. Seborrheic dermatitis (B) is usually more erythematous and has a thicker scale than tinea versicolor. Furthermore, seborrheic dermatitis typically causes lesions on the scalp, eyebrows, and nasolabial folds. Vitiligo (D) can be distinguished from tinea versicolor because it causes depigmentation rather than hypopigmentation.

A 28-year-old pregnant woman presents to the emergency department with paroxysms of right flank pain associated with hematuria, nausea, and vomiting. Physical examination reveals right costovertebral angle tenderness. Which of the following is the most appropriate diagnostic test for this patient? A. CT scan B. KUB radiograph C. Ultrasonography D. Urinalysis

Correct Answer: (C) Ultrasonography Explanation: Nephrolithiasis is a renal disorder characterized by stone formation associated with renal colic and hematuria. Approximately 80% of kidney stones are caused by calcium composed primarily of calcium oxalate that arises in the renal medullary interstitium or less often of calcium phosphate that forms the Randall plaque. Other types of kidney stones include uric acid (acidic urine pH < 5.5), struvite (alkaline urine and presence of urease-producing Proteus or Klebsiella), and cystine stones. Risk factors for nephrolithiasis include low fluid intake, history of prior nephrolithiasis, family history of nephrolithiasis, enhanced enteric oxalate absorption due to malabsorption (such as after a gastric bypass or bariatric surgery), hyperparathyroidism, hypercalciuria, increased animal protein intake, diabetes, gout, hypertension, medications (such as acyclovir, indinavir, and sulfasalazine), and increased urinary oxalate. Patients present with waxing and waning pain that may be mild, moderate, or severe and develops in paroxysms or waves. The site of stone obstruction determines the location of pain. Hence, upper ureteral or renal pelvic obstruction may lead to flank tenderness, whereas lower ureteral obstruction causes pain to radiate to the ipsilateral testicle or labium. A variable location of pain may mimic dissecting aneurysm or an acute abdomen. Gross or microscopic hematuria may or may not be present but is the single most discriminating predictor of nephrolithiasis in patients who present with unilateral flank pain. Nausea, vomiting, dysuria, and urgency may accompany flank pain and hematuria. Nephrolithiasis may lead to persistent renal obstruction that may cause permanent kidney damage if left untreated. Physical examination may reveal positive costovertebral angle tenderness. Urinalysis may show gross or microscopic hematuria and acidic or alkaline pH. For patients suspected of having hyperparathyroidism or gout, serum parathyroid hormone or uric acid levels should be obtained. Initial diagnosis of nephrolithiasis is either by kidney, ureter, and bladder radiograph or ultrasound. Ultrasound is the preferred imaging modality in pregnant patients and children with suspected kidney stones. CT scan of the abdomen and pelvis without contrast using low-dose radiation scanning protocols is the preferred diagnostic tool for adults suspected of having nephrolithiasis. Treatment is supportive until the stone passes and involves pain medication (nonsteroidal anti-inflammatory drugs and opioids), IV hydration, and urological consultation. Stones less than 5 mm pass spontaneously while stones that are 6-10 mm require medical expulsive therapy using pain medication and intravenous hydration. Alpha-blockers, such as tamsulosin, may facilitate stone expulsion. Stones > 10 mm may require extracorporeal shockwave lithotripsy or ureteroscopy stone extraction. Urology consultation is needed in patients with urosepsis, acute kidney injury, anuria, and intractable pain, nausea, or vomiting. Patients should be educated on the importance of adequate hydration. They should also be encouraged to strain their urine and to bring in any stone that passed. While computed tomography scan (A) is the preferred imaging modality to diagnose nephrolithiasis in adults, it is associated with teratogenicity in pregnant individuals. Kidney, ureter, and bladder radiograph (B) may show kidney stones but is not indicated in pregnant women. Gross or microscopic hematuria on urinalysis (D) may suggest nephrolithiasis. However, in a pregnant patient, ultrasonography is required to confirm the diagnosis.

A 63-year-old man with poorly controlled diabetes presents to the emergency department with altered mental status, fever, and frontal headache. He has been taking analgesics regularly without any improvement in his pain. He is febrile on examination. He appears lethargic and is noted to have right-sided hemiparesis. He is rushed to CT for STAT imaging of the head where he has a grand mal seizure. Postcontrast imaging reveals a large ring-enhancing lesion in the left frontal lobe with surrounding cerebral edema. What would you expect to learn from his history based on the likely diagnosis? A. He has a history of radiation to the brain B. He recently traveled outside the country C. He was recently treated for acute mastoiditis D. He was recently treated for acute sinusitis

Correct Answer: (D) He was recently treated for acute sinusitis Explanation: Infection of the sinuses can result in brain abscess within the frontal lobes. Brain abscess is a collection of bacteria within the brain as a result of direct spread of bacteria from one site or through seeding via the spread of bacteria through the blood. Direct spread can include infections such as otitis media, mastoiditis, frontal sinusitis, ethmoid sinusitis, and dental infections. Foreign bodies secondary to trauma (such as bullets) and neurosurgical procedures can also precipitate abscess formation. Hematogenous spread includes infections that originate elsewhere, such as from the lungs, skin, pelvis, and heart. Brain abscess occurs more commonly in the frontal and temporal lobes. The most frequent bacteria implicated in brain abscesses are Streptococcus viridans, Streptococcus milleri, and Staphylococcus aureus. Patients at increased risk are those who are immunocompromised. Symptoms can include headache (typically at the side of the abscess), change in mental status, and fever. Physical examination can demonstrate focal neurological deficits. Seizures may be the first presenting sign and are often common in those with frontal abscesses. A CT scan of the head is often first performed, however, MRI is more sensitive for the diagnosis and should be performed with gadolinium. Imaging can detect cerebritis, central necrosis, ring-enhancing lesions, and cerebral edema. While patients may present with symptoms suggestive of meningitis, a lumbar puncture should not be performed due to the risk of cerebral herniation. The abscess can be cultured via CT-guided aspiration or surgical intervention. Aspiration is preferred to surgical excision. Antibiotic choice is dependent on probability of pathogen involved based on the patient's history. Drug choice often includes penicillin G, metronidazole, ceftriaxone, ceftazidime, or vancomycin. Empiric therapy is dependent on the source. If the bacteria originates from an oral bacteria, the patient should be started on metronidazole with either penicillin G or ceftriaxone. For a patient with an abscess stemming from a sinus or otogenic infection, the patient should be started empirically on metronidazole with cefotaxime. If the source is unknown, vancomycin should be added. Glucocorticoids are considered if mass effect is present on imaging. He has a history of radiation to the brain (A) is incorrect. Prior neurosurgical intervention would be a risk for abscess formation. He recently traveled outside the country (B) is incorrect. This would not precipitate his development of brain abscess. He was recently treated for acute mastoiditis (C) is incorrect. A recent acute mastoiditis infection would reveal brain abscess development in the temporal lobe, not the frontal lobe.

A 42-year-old man reports to the clinic complaining of tenderness and swelling over his elbow. He noticed the area the day after he hit his elbow on a door frame. He has a history of asthma, and he takes fluticasone daily. Examination of the joint reveals an oval-shaped swelling over the olecranon process of the ulna without erythema, warmth, or overlying skin lesions. Which of the following is the best initial treatment for this patient? A. Aspiration B. Elbow brace locked in full extension C. Glucocorticoid injection D. Ibuprofen

Correct Answer: (D) Ibuprofen Explanation: Olecranon bursitis results from inflammation of the bursal sac that overlies the posterior olecranon process of the ulna. This inflammation can occur secondary to trauma, hemorrhage, sepsis, or arthritic conditions, such as gout, rheumatoid arthritis, or osteoarthritis. It commonly causes focal tenderness and an oval swelling at the tip of the elbow. Joint motion is not affected, and extension of the elbow does not accentuate pain. Patients may report tightness or pain with end-range flexion of the joint. Septic bursitis may be indicated with overlying tenderness, erythema, warmth, cellulitis, or skin lesions. Imaging of the area is not indicated in traumatic olecranon bursitis, however, physical exam should evaluate for any signs of complications, such as overlying breaks in the skin or signs of infection. The bursa should be aspirated if infection is suspected or to diagnose a microcrystalline disorder, such as gout. Aspirated fluid should undergo cell count, Gram stain, and culture. The primary treatment for olecranon bursitis is joint protection with elbow pads, sleeves, and orthoses. This treatment, however, should not impede normal range of motion of the joint. Patients with underlying microcrystalline conditions should be treated accordingly with analgesics and appropriate urate lowering therapies, such as colchicine. Infectious cases require prompt initiation of antibiotic therapy (clindamycin, dicloxacillin) and drainage of the infected bursal sac. Therapeutic fluid aspiration may alleviate pressure in patients with aseptic bursitis, but repeated aspiration is not recommended as the fluid will commonly reaccumulate regardless, and the procedure introduces trauma and predisposes the patient to infection. Anti-inflammatory agents (ibuprofen, meloxicam) can be used in those without contraindications. Glucocorticoid injection, while helpful in treating other bursitis syndromes, is not recommended for patients with olecranon bursitis as it can cause skin atrophy and has not been clinically proven to be beneficial. Chronic, persistent olecranon bursitis may require arthroscopic surgical removal of the bursal sac. Aspiration (A) of the inflamed bursa is not necessary in this patient with aseptic olecranon bursitis secondary to trauma. This is more useful in the diagnosis of suspected septic bursitis, which presents with fever, pain, tenderness, erythema, and warmth. The patient should not be given an elbow brace locked in full extension (B) as this reduces his range of motion and the functional ability of the elbow joint. Elbow orthoses and pads that do not hinder motion can be prescribed. Glucocorticoid injection (C) is not an initial treatment of choice in patients with olecranon bursitis as it has not been proven beneficial and predisposes the patient to infection, skin atrophy, and other adverse side effects of glucocorticoid medications. It is useful in bursitis of the subacromial, anserine, medial collateral ligament, and trochanteric bursae.

A 75-year-old woman with chronic kidney disease presents to the emergency department with dyspnea and chest pain on inspiration. She underwent a surgical repair of a left hip fracture she sustained four days ago and has not been ambulatory since the surgery. Physical examination reveals she is tachycardic and tachypneic. Laboratory studies show elevated D-dimer but are otherwise normal. ECG reveals sinus tachycardia. Which of the following is the most likely diagnosis? A. Heart failure B. Pericarditis C. Pneumonia D. Pulmonary embolism

Correct Answer: (D) Pulmonary embolism Explanation: Pulmonary embolism (also called pulmonary venous thromboembolism) is a common, potentially fatal complication of venous thrombus formation within the deep venous circulation. Substances that may embolize in the venous circulation include air, parasite eggs, amniotic fluid, fat, foreign bodies, septic emboli, and tumor cells. Pulmonary emboli develop in approximately 50-60% of patients with proximal deep venous thrombosis. Patients with symptomatic pulmonary emboli (50-70%) will have lower extremity deep vein thrombosis when evaluated. Hence, pulmonary embolism and deep venous thrombosis are two manifestations of the same disease. Risk factors for pulmonary embolism, known as the Virchow triad, include venous stasis (postsurgery, bed rest, obesity), hypercoagulable state (pregnancy, polycythemia), and vascular injury (surgery, oral contraceptives, hormonal therapy, hyperhomocysteinemia, antiphospholipid antibodies). Clinical manifestations of pulmonary embolism are nonspecific, making diagnosis challenging. Patients typically present with dyspnea, chest pain on inspiration, or tachypnea. An ECG is typically abnormal in 70% of patients with pulmonary embolism and may show sinus tachycardia, nonspecific ST-T wave changes, right bundle branch block, right axis deviation, and P pulmonale. S1Q3T3 morphology may be noted on ECG but is nonspecific. Arterial blood gas may reveal respiratory alkalosis due to hyperventilation. Plasma levels of D-dimer may be elevated (> 500 ng/mL) in patients with pulmonary embolism but may also be used to exclude the diagnosis of pulmonary embolism in patients with normal levels. Therefore, low plasma levels of D-dimer have a high negative predictive value. A chest radiograph is needed to exclude other common pulmonary diseases and will also show nonspecific findings including Hampton hump (pleural-based areas of increased opacity) and Westermark sign (prominent central pulmonary artery with local oligemia). A normal chest radiograph in the setting of hypoxemia should increase the clinical suspicion of pulmonary embolism. Helical computed tomography pulmonary angiography is the test of choice in the setting of moderate-to-high clinical suspicion of pulmonary embolism. A ventilation-perfusion lung scan may be used in patients with chronic renal disease who cannot tolerate the contrast dye used in computed tomography angiography or in pregnant patients with high suspicion of embolism to reduce the radiation dose to the fetus. Venous ultrasound is the test of choice to detect proximal deep venous thrombosis. Treatment is anticoagulation (using heparin, low-molecular-weight heparins), thrombolysis for hemodynamically unstable patients (streptokinase, urokinase), inferior vena cava filter (for patients who have major contraindications to anticoagulation or recurrent thromboembolism), or surgical extraction/pulmonary embolectomy as a last resort with potential fatal complications. Prognosis of patients with diagnosed and appropriately treated pulmonary embolism is generally good. Prevention is the key to minimize the mortality rate of pulmonary embolism. Therefore, patients undergoing hip fracture surgery should receive prophylactic therapy using anticoagulation. Individuals with heart failure (A) may present with symptoms such as dyspnea and leg edema. Lung auscultation typically reveal crackles and chest radiograph may show blunting of the costophrenic angles. The pain of pericarditis (B) is generally pleuritic and the typical ECG finding is diffuse ST-segment elevation and PR depression. Fever, lobar consolidation on chest radiograph, and leukocytosis favor the diagnosis of pneumonia (C) over pulmonary embolism.

A 26-year-old man presents to the emergency room with a superficial hand wound. According to the patient, he was attempting suicide and the gun backfired. The patient reports depressed mood all day long for the past three weeks, insomnia, weight loss, fatigue, thoughts of worthlessness, and suicidal ideation. He takes no medications, has never been hospitalized before, and has a negative urine drug screen. Which of the following options represents the best initial treatment for this patient upon admission to the psychiatry unit for monitoring? A. Electroconvulsive therapy B. Hypnotherapy C. Ketamine injection D. SNRI

Correct Answer: (D) SNRI Explanation: Major depressive disorder is characterized as a mood disorder, presenting as persistent depressed mood or anhedonia for at least two weeks duration. Women are more likely than men to experience depression, although men are more likely than women to die by suicide. Risk factors for major depressive disorder include family history, stressful life events, substance use disorder, and social isolation. For diagnosis of major depressive disorder, in addition to persistently depressed mood for at least two weeks, the patient must also exhibit five or more of the following symptoms: hypersomnia, insomnia, appetite changes, significant weight change, psychomotor retardation or agitation, fatigue, low energy, impaired concentration, impaired decision-making skills, thoughts of worthlessness or excessive guilt, recurrent thoughts of death, suicidal ideation, or suicide attempts. The patient must also have no previous episodes of mania or hypomania for a diagnosis of major depressive disorder, and other medical causes of depressed mood should be ruled out (such as mood-altering drug use, hypothyroidism, hypercalcemia, or autoimmune disorders). First-line treatment of major depressive disorder is with selective serotonin reuptake inhibitors or selective norepinephrine reuptake inhibitors. The latter class has proven slightly more efficacious. Severely depressed or suicidal individuals should be hospitalized during initiation of treatment, as increased suicidality in the early stages of treatment is a side effect of this class of medication. The addition of psychotherapy to pharmacotherapy is more effective for treatment of major depressive disorder than either two modalities alone. Treatment should be continued for four to nine months, at which time the patient and provider can reevaluate treatment and determine whether further continuation of pharmacotherapy is warranted. If medication is discontinued, dosage should be tapered first. Recurrence is common, and continuation of treatment indefinitely is recommended for those who have had more than one recurrence. Electroconvulsive therapy (A) is an acceptable choice of treatment for patients with severe or persistent major depressive disorder. In a patient who is treatment-naive, pharmacotherapy should be administered first, with electroconvulsive therapy a second-line option for those who fail, or have an inadequate response to, pharmacotherapy. Electroconvulsive therapy is generally safe and effective, although the risk of relapse is high. Hypnotherapy (B) has not been shown to be effective in the treatment of major depressive disorder. Other forms of psychotherapy, including the cognitive behavioral approach, are more effective. Ketamine injection (C) is a relatively new modality for the treatment of refractory major depressive disorder and is still under investigation.

A 33-year-old woman presents with recent-onset fever, abdominal cramps, and diarrhea. She describes frequent, small-volume, bloody diarrhea and reports no recent antibiotic use or history of gastrointestinal disorders. Vitals reveal HR 105 bpm, RR 19/min, BP 138/78 mm Hg, T 103.9°F, and SpO2 98% on room air. Which one of the following suspected organisms requires empiric antibiotic therapy? A. Clostridium perfringens B. Salmonella C. Shiga-toxin-producing Escherichia coli D. Shigella

Correct Answer: (D) Shigella Explanation: Acute infectious diarrhea is defined as a diarrheal illness that lasts < 2 weeks and is caused by a virus, bacteria, or protozoan. Common causes of infectious diarrhea include Salmonella, Campylobacter, Shigella, Cryptosporidium, Shiga-toxin-producing Escherichia coli, Yersinia, Entamoeba histolytica, and Vibrio. Diarrhea can be severe or bloody, implicating bacterial pathogens, or watery, often caused by viral agents or protozoa. Shigella is a particularly virulent bacterium that causes dysenteric diarrhea and is associated with significant morbidity and mortality worldwide. It is spread via the fecal-oral route in industrialized countries and direct person-to-person contact or from contaminated food or water in resource-limited countries. Shiga toxin, produced by some strains of Shigella and E. coli, may result in hemolytic uremic syndrome in children. Common presentations of patients with Shigella gastroenteritis include high fever, abdominal cramps, tenesmus, and bloody mucoid diarrhea. Complications associated with this condition include proctitis, rectal prolapse, toxic megacolon, intestinal obstruction, and colonic perforation. Early diagnostic testing measures include microscopic evaluation of the stool, revealing white and red blood cells. Stool culture is the diagnostic test of choice, and all isolates should undergo susceptibility testing to ensure proper antibiotic selection. First-line treatment of Shigella-related diarrhea is supportive, with intravenous hydration indicated only if the patient cannot tolerate oral rehydration strategies. Antimotility agents (e.g., diphenoxylate, paregoric, loperamide) should be avoided as they may prolong fever, diarrhea, and bacterial shedding in the stool. Empiric antibiotic therapy is indicated only in symptomatic patients while awaiting confirmation of the diagnosis. In this subset of patients, antimicrobial agents have been shown to reduce the duration of fever and diarrhea by about 2 days and shorten the duration of bacterial shedding, which reduces the risk of person-to-person transmission. While antibiotic selection should be dictated by susceptibility reports, most regimens include a fluoroquinolone (levofloxacin, ciprofloxacin), azithromycin, a third-generation cephalosporin (cefixime, ceftriaxone), trimethoprim-sulfamethoxazole, or ampicillin. Oral rehydration using glucose-based formulations is important to replete fluids and electrolytes. Strategies for preventing transmission include frequent handwashing and avoidance of intercourse until 2 weeks after the diarrhea resolves. Conversion to negative stool cultures is usually evident after 48 hours of antimicrobial therapy, at which time patients who handle food can return to food preparation. Clostridium perfringens (A) infection does not require antibiotic therapy. It presents with an abrupt onset of profuse watery diarrhea, abdominal cramps, nausea, and occasional vomiting, and patients typically recover within 24-48 hours with supportive care. Salmonella (B) is contracted through the consumption of contaminated eggs, poultry, unpasteurized milk, cheese, juices, and raw fruits and vegetables. It may cause fever, nausea, and vomiting, and the diarrhea is typically not bloody. Empiric antibiotic treatment is not generally indicated for gastroenteritis caused by this pathogen. Shiga-toxin-producing Escherichia coli (C) belongs to the enterohemorrhagic strains of E. coli and is contracted primarily from undercooked beef. It is a self-limited disease in adults that does not require antibiotic therapy, which is associated with an increased risk of developing hemolytic uremic syndrome.

A 23-year-old man presents to the emergency department with a gradually worsening dry cough, low-grade fever, and intermittent dyspnea for three weeks. Physical examination reveals rales, wheezing, and rhinorrhea. Vital signs are temperature 100.1 °F, respiratory rate 22, blood pressure 123/65 mm Hg, and oxygen saturation 93%. Sinus percussion tenderness is absent. A chest radiograph reveals bilateral interstitial infiltrates. Gram stain of the sputum did not reveal any organism. Which of the following is the most likely diagnosis? A. Chlamydophila pneumoniae pneumonia B. Chlamydophila psittaci pneumonia C. Mycoplasma pneumoniae pneumonia D. Streptococcus pneumoniae pneumonia

Correct Answer: (D) Streptococcus pneumoniae pneumonia Explanation: Mycoplasma pneumoniae is the smallest free-living bacterium that causes upper respiratory infections, acute bronchitis, and atypical community-acquired pneumonia. Mycoplasma pneumoniae possesses a specialized tip organelle that produces a P1 adhesin protein, which is a protein molecule that facilitates adherence and gliding motility along the respiratory epithelium. Mycoplasma pneumoniae infections are transmitted via respiratory droplets with an incubation period of approximately two to three weeks. Community-acquired pneumonia caused by Mycoplasma pneumoniae is more common in young children than in adolescents and adults. Patients present with a gradual onset of productive or nonproductive cough, headache, malaise, sore throat, low-grade fever, pleuritic chest pain, and shortness of breath. Other nonrespiratory manifestations of Mycoplasma pneumoniae include mild hemolysis, encephalitis, urticaria, erythema multiforme, Stevens-Johnson syndrome, and pericarditis. Physical examination may reveal rales, wheezing, cervical lymphadenopathy, rhinorrhea, and otitis media. Microbiologic diagnosis of Mycoplasma pneumoniae is not necessary in the majority of patients with community-acquired pneumonia as empiric treatment regimes typically include an agent that targets atypical organisms such as Mycoplasma pneumoniae. In the setting of severe pneumonia or in patients with clinical features suggestive of Mycoplasma pneumoniae infection, chest radiograph, laboratory studies, and polymerase chain reaction-based assays should be obtained. Chest radiograph may reveal unilateral or bilateral reticulonodular opacities, thickened bronchial shadow, interstitial infiltrates, or areas of atelectasis. Laboratory studies may reveal signs of hemolysis including anemia and elevations in unconjugated bilirubin, lactate dehydrogenase, and reticulocyte count. Gram stain is generally not helpful in diagnosis of Mycoplasma pneumoniae since the organism lacks a cell wall and is not visible on Gram stain. Empiric treatment of community-acquired pneumonia caused by Mycoplasma pneumoniae includes azithromycin or doxycycline for outpatients. Respiratory fluoroquinolone (levofloxacin, moxifloxacin) or a beta-lactam plus macrolide or doxycycline is used for outpatients with comorbidities, immunocompromised patients, or patients with recent antibiotic use. Beta-lactam (ceftriaxone) or respiratory fluoroquinolone plus a macrolide or doxycycline is used for hospitalized patients. For patients with a microbiologically-confirmed Mycoplasma pneumoniae infection, treatment includes a macrolide (azithromycin), a tetracycline (doxycycline), and a respiratory fluoroquinolone (levofloxacin). Hoarseness and sinus percussion tenderness are more common in patients with Chlamydophila pneumoniae pneumonia (A) than in Mycoplasma pneumoniae pneumonia. A chest radiograph typically shows lower lobe single subsegmental infiltrate. History of exposure to birds is an important diagnostic clue to the diagnosis of Chlamydophila psittaci pneumonia (B). Physical examination findings in Chlamydophila psittaci pneumonia include pulse-temperature dissociation, splenomegaly, and somnolence. Chest radiograph classically show single lower lobar consolidation. Individuals with Streptococcus pneumoniae pneumonia (D) usually present with acute or subacute fever, cough, and dyspnea. In these patients, a chest radiograph typically shows lobar consolidation and sputum Gram stain reveals Gram-positive diplococci.

In a patient with subarachnoid hemorrhage, what develops as a result of lysis of the subarachnoid blood clots and release of nitrous oxide and endothelin approximately three days after onset of symptoms? A. Increased intracranial pressure B. Meningismus C. Seizure D. Vasospasm

Correct Answer: (D) Vasospasm Explanation: Subarachnoid hemorrhage is a type of hemorrhagic stroke often resulting from a ruptured saccular aneurysm. Vasospasm is a common complication of subarachnoid hemorrhage, occurring no earlier than three days after the onset of hemorrhage. It can reach its peak at day 7 or 8. This is due to the substances released as a result of the lysis of blood products, primarily nitrous oxide and endothelin. Risk factors for subarachnoid hemorrhage include hypertension, cigarette smoking, family history of subarachnoid hemorrhage, cocaine use, polycystic kidney disease, and Ehlers-Danlos syndrome. The presence of blood in the intraventricular space causes increased intracranial pressure. Patients will present with sudden onset of a severe headache describing it as the "worst headache of my life." Other associated symptoms include loss of consciousness, nausea, vomiting, and meningismus. Meningismus are symptoms resulting from meningeal irritation, primarily headache and photophobia. Noncontrast CT scan of the head should be performed emergently, which may or may not show bleeding in the subarachnoid and intraventricular space. If the CT scan is negative, a lumbar puncture must be performed in patients who are suspected of having a subarachnoid hemorrhage. Lumbar puncture reveals hemoglobin degradation products (xanthochromia) and elevated red blood cells that do not diminish with each collection tube. Once the diagnosis has been confirmed, angiography should be performed to evaluate the underlying etiology of the hemorrhage. An aneurysm will often need coiling or clipping. Elevated intracranial pressure may be treated with ventriculostomy or with medical management (hypertonic saline or mannitol). Severity of subarachnoid hemorrhage is graded based on the Hunt and Hess grading system. Complications can include rebleeding, hydrocephalus, seizures, vasospasm, and death. Vasospasm is associated with significant morbidity and mortality, which is due to cerebral hypoperfusion and delayed cerebral ischemia. Vasospasm is treated with calcium channel blockers, primarily nimodipine. Increased intracranial pressure (A) is incorrect. This may be a complication of subarachnoid hemorrhage but does not occur due to the release of nitrous oxide and endothelin. Meningismus (B) is incorrect. This is a common presenting symptom of subarachnoid hemorrhage. Seizure (C) is incorrect. It does not occur due to this mechanism, but can occur as a result of elevated intracranial pressure and cortical injury.

Acute Cholangitis

Patient presents with right upper quadrant pain, jaundice, fever (Charcot triad) Diagnosis is made by RUQ ultrasound, CT scan, or ERCP (gold standard) Most commonly caused by choledocholithiasis that leads to bacterial infection, E. coli Treatment is antibiotics, definitive treatment is ERCP with antibiotics typically an adjunct Charcot triad + hypotension and AMS = Reynolds pentad, acute obstruction

Primary Immune Thrombocytopenia

Patient will be a child 2-6 years old History of recent viral infection Red spots on skin or easy bleeding PE will show petechiae, purpura, and gingival bleeding Labs will show platelets < 100,000/µL Most commonly caused by antiplatelet antibodies Treatment is observation, steroids, IVIG

Corneal Ulcer

Patient will have a history of trauma, incomplete closure, or extended contact lens use PE will show oval ulcer with ragged edges, severe conjunctival inflammation Most commonly caused by Staphylococcus, Pseudomonas (contact lens wearers), Streptococcus pneumoniae Treatment is emergent ophthalmology consult

Mitral Regurgitation

History of ischemic heart disease, endocarditis, MI, trauma Presentation: Acute Sx: dyspnea, pulmonary edema, cardiogenic edema Chronic Sx: asymptomatic until CHF onset, S3 PE: Acute: unique, harsh, midsystolic murmur best heard at apex that radiates to the base rather than the axilla Chronic: blowing holosystolic murmur best heard at apex with radiation to axilla Diagnosis is made by echo Treatment: Acute: nitroprusside, dobutamine, intra-aortic balloon pump, emergency surgery Chronic: CHF Rx, valve repair or replacement

Aplastic Anemia

History of recurrent infections Mucosal hemorrhage, prolonged menstrual bleeding, petechiae, ecchymoses, and fatigue Labs will show peripheral pancytopenia and bone marrow hypoplasia or aplasia Diagnosis is made by bone marrow aspiration and biopsy

Knee Dislocation

History of violent trauma PE will show dimple sign with posterolateral dislocation Most common causes: Anterior dislocation: hyperextension of the knee Posterior dislocation: a direct blow to the anterior tibia with the knee flexed Treatment is reduction and knee splint with 15-20 degrees of flexion Complications can involve popliteal artery and peroneal nerve

Which muscle tendons are involved in de Quervain tenosynovitis?

The abductor pollicis longus and the extensor pollicis brevis.

What causes tactile fremitus, dullness to percussion, and egophony?

The accumulation of white blood cells, fluid, and protein in the alveolar space.

What is Terson syndrome?

The combination of vitreous hemorrhages and subarachnoid hemorrhage.


Kaugnay na mga set ng pag-aaral

Unit 1, Period 1: Native American Societies before European Contact through Permanent English Settlement, 1491 - 1607

View Set

ACCT 308 Exam 1 Homework Questions

View Set

T2 Cumulative Exam (Unit 1, 2, 3, 4, 5, 6, 7)

View Set